Download as docx, pdf, or txt
Download as docx, pdf, or txt
You are on page 1of 67

CONSTITUTIONAL LAW LECTURE

NOTES

CHARACTERISATION+INTERPRETATION
(LECTURE 1)
 
 

INTERPRETATION
The significance of the Engineers Case for interpretation of the Cth Constitution
 Judges no longer interpret the Constitution importing external (cf internal)
principles (in particular – no more reserved power doctrine)
 Constitution is interpreted giving words their natural meaning in context
o Textualism: giving words of the Constitution their natural meaning
unconstrained by unnecessary implications or preconceived ideas about
the federal balance
o Legalism: judges should make their decisions by reference to legal sources
and materials (not external sources)
 This has expanded the scope of Cth powers
o E.g work choices
 Critics
o It is positive: a more stable method of interpretation that doesnnt allow for
importation of judicial values
o It is negative: it has destroyed the federal distribution of powers the
drafters of the constitution intended

What is interpretation? 

 In Commonwealth v Tasmania (1983) 158 CLR 1 Mason J distinguished between


interpretation and characterisation as follows:
o ‘[There are] two distinct and separate questions: (1) what is the scope of the
power; and (2) is the law in truth a law with respect to the subject matter of the
power, once its scope has been ascertained. Characterization, the name given
to the process of arriving at an answer to the second question, cannot begin
until the first question is answered’ (88).

The first question – interpretation – therefore involves determining the scope of


Commonwealth power as set out in the Constitution, especially in sections 51 and 52.
 In the early years of the Australian Cth, the ‘reserved State powers’ doctrine dictated
that the grants of law-making power given to the Ch – the scope of the power - should
be narrowly interpreted so as not to encroach on the traditional powers of the States
o A narrow interpretation that would limit powers of parliament legislature and
maximise states legislative powers 
 See R v Barger (1908) for example: 
The Excise Tariff Act 1906 (Cth) imposed a tax on manufacturers of agricultural implements.
Section 2 exempted goods from the tax if they had been manufactured under conditions
declared by Parliament to be fair and reasonable or in accordance with a federal industrial
award.
Nevertheless, the outcome: legislation was found to be invalid. HC (3:2
majority) found that the s 51(ii) Taxation power should be given a narrow
interpretation and shouldn’t be interpreted in such a way that allowed the
commonwealth to ‘directly regulate the domestic affairs of the states’
o Isaacs J (dissenting): no derogations from express grant, except those which
are expressly stated or which of necessity inhere.

 However, this doctrine was ‘exploded in Engineers Case (1920) 


o Facts: Industrial dispute involving an engineers trade union and various
government employees
o Issue: whether the Commonwealth Parliament could pass legislation binding
state employers under one of the heads of power (the conciliation and
arbitration power)
o Majority: the HoP should be interpreted in accrd with the natural meaning of
the words, not subject to implied limitations unless these follow necessarily
from the text.
o Textualism, Legalism

 Jumbunna Principle
o ‘...where the question is whether the Constitution has used an expression in the wider or
in the narrower sense, the Court should, in my opinion, always lean to the broader
interpretation unless there is something in the context or in the rest of the Constitution to
indicate that the narrower interpretation will best carry out its object and purpose’
The dual effect of the Engineers Case and the Jumbunna principle is that the
Commonwealth’s powers are broadly interpreted:
o ‘A power to make laws “with respect to” a specific subject is as wide a
legislative power as can be created. No form of words has been suggested
which would give a wider power. The power conferred upon a Parliament by
such words in an Imperial statute is plenary – as wide as that of the Imperial
Parliament itself ... But the power is plenary only with respect to the specified
subject’: Bank Nationalisation Case per Latham CJ).

CHARACTERISATION
What is characterisation? 
 Characterisation is the process of determining whether a Cth law can be described as
a law ‘with respect to’ a head of Cth power 
Cth law will prevail over state law if there is any inconsistency 
o
States and territories can legislate areas that the parliament have not been
o
given powers to legislate on 
Heads of Power in the Commonwealth Constitution (Section 51) 
 Entrust certain law-making powers in the Commonwealth 
 Section 51 and 52 accord certain law-making powers to the Commonwealth
Parliament 
 The Parliament is authorized to make laws ‘with respect to’ these powers (no duty on
the commonwealth to use this power, they are just authorised)
 The States may legislate within the areas covered by section 51 (but not section 52)
subject to section 109 (inconsistency) [where there is inconsistency, the commonwealth
law takes priority over state law]
Subject Matter v Purpose Powers 
The HC divides the HoP in sections 51 and 52 into ‘subject matter’ and ‘purpose’

powers 
 A different test of characterisation is applied depending upon how the HoP is
categorised 
 For subject matter HoP a ‘sufficient connection’ test is applied 
 For purpose HoP a ‘proportionality’ test is applied 
 The ‘purpose’ powers so far identified by the HCC are: the ‘defence power’
(s51(vi)) and the ‘external affairs’ power (s51(xxix) 

Single Characterisation 
 Characterisation was once guided by a search for the ‘pith and substance’, or ‘single
character’ of the law
 (R v Barger (1908)), the majority held that the purpose of the Excise Tariff Act
1906 (Cth) was ‘not to raise money for the purposes of government, but to
regulate the conditions of labour’ (74):
o Good example of single characterisation, the HC said need to interpret
the scope of the legislation narrowly and then need to identify the
single character of the legislation – not a law about taxation, instead
conditions of labour, therefore does not fall within s90 (excise powers) 

Dual Characterisation 
 Characterisation: process of interpreting Cth law
 Cth can have more than one subject matter/purpose. (e.g. a speed lmit could be characterised
as aa law about road safety or a law about cars)
o Subject matter:
 What the law is doing
 The rights, duties, powers and privileges which the law changes, regulates or
abolishes (Fairfax, Kitto J; First Uniform Tax, Latham CJ)
 A law will be a law ‘with respect to’ a subject matter if it has a sufficient connection to that
subject matter (Re Dingjan, McHugh J)
 Has the effect of expanding Cth legislative power:
o Cth has enumerated powers (law must be capable as being characterised as falling
within a Cth HoP)
o States have residual powers (the leftovers)
o Cth ultimately has control over areas within the scope of it’s HoP’s because of s 109

Fairfax (1965) 114 CLR 1:


 The issue was whether section 11 of the Income Tax and Social Services Contribution
Assessment Act 1961 (Cth) was validly enacted under the Commonwealth’s taxation
power in section 51(ii) of the Constitution.
 Section 11 inserted an amendment that could be used to induce the trustees of
superannuation funds to invest in Commonwealth bonds by exempting the funds from
income tax if they did and by subjecting them to a special rate of taxation if they did
not.
o Kitto J: ‘But it is by no means a settled doctrine that a law which purports to
provide for a tax upon behaviour is in substance not a law with respect to
taxation if it exhibits on its face a purpose of suppressing or discouraging the
behaviour and is to be explained more convincingly as a means to that end
than as a means to provide the Government with revenue’ (11).
o Kitto cited the dicta from Dixon J from Melbourne Corporation with
approval: ‘Speaking generally, once it appears that a federal law has an
actual and immediate operation within a field assigned to the
Commonwealth as a subject of legislative power, that is enough ... That it
discloses another purpose and that purpose lies outside the area of federal
power are considerations which will not in such a case suffice to invalidate
the law’

Murphyores Incorporated Pty Ltd v Cth (1976) 


 About the trade and commerce power in s51(i). The Customs Act 1901 (Cth) gave
power under s112 to make regulations prohibiting exports. Regulation 9(3) provided
that the approval of the Minister of State for Mineral and Energy was required for
certain exports 
Facts
Section 112 of the Customs Act 1901 prohibited the exportation of mineral sands unless
authorised by the Minister. Murphyores Inc Pty Ltd, which held leases from the state
of Queensland to mine mineral sands on Fraser Island, sought permission from the Minister
to export mineral sands. Such authorisation was withheld pending the outcome of an
environmental inquiry. Murphyores challenged the constitutional validity of prohibition and
sought an injunction to the study, and a declaration that the Minister cannot make a
prohibition for environmental purposes.
 Issue: was this law constitutional
 In a unanimous decision, the court held the legislation was a valid exercise of the
trade and commerce power. Section 51(i) was a non-purposive power, and the only
relevant factor was the subject matter of trade and commerce. The motive and purpose
behind the legislation was irrelevant.
 T&C is a subject matter power, allows for regulation of foreign/interstate T&C
activities.

The law can have multiple characters, only one needs to relate to the head of power – how do
you connect the two – sufficient connection test

Sufficient Connection Test 

 Re Dingjan per McHugh J: ‘If a connection exists between the law and a s 51 HoP,
the law will be “with respect to” that HoP unless the connection is, in the words of
Dixon J [Melbourne Corporation], “so insubstantial, tenuous or distant” that it cannot
sensibly be described as a law “with respect to” the HoP’
 Grain Pool
 First, the constitutional text is to be construed “with all the generality which the words
used admit” 
 Secondly, the character of the law in question must be determined by reference to the
rights, powers, liabilities, duties and privileges which it creates 
 Thirdly, the practical as well as the legal operation of the law must be examined –
what is the law doing 
 Fourthly, it will be valid notwithstanding that there is no independent connection
between the two subject matters (one within s51 and one fall out s51)
 Finally, if a sufficient connection with the HoP does exist, the justice and wisdom of
the law, and the degree to which the means it adopts are necessary or desirable, are
matters of legislative choice 

Interpretation and characterisation


What is the scope of the head of power?
What is the character of the law?
Is there a sufficient connection between the head of power and the law?

Reading down, severance and invalidity

Unconstitutional statutory provisions may be ‘read down’: ‘If the impugned statute is capable
of bearing two meanings, one of which would render it invalid and the other which would
render it valid, the latter is to be preferred’ (Public Service Association of SA per Heydon J).

If unconstitutional statutory provisions cannot be read down, it may be possible for them to
be ‘severed’. Alternatively, the statute as a whole may be invalid.

A finding of invalidity will normally mean that the statutory provision is void ab initio i.e.
void from its inception.

The techniques of reading down and severance accord with s 15A of the Acts Interpretation
Act 1901 (Cth):

‘Every Act shall be read and construed subject to the Constitution, and so as not to exceed the
legislative power of the Commonwealth, to the intent that where any enactment thereof
would, but for this section, have been construed as being in excess of that power, it shall
nevertheless be a valid enactment to the extent to which it is not in excess of that power.’

HoP problem question


1. Who am I advising and what do they want?
Very short, to orientate your answer
2. Scope of power
Cth power interpreted broadly-Jumbanna
The High Court has held that each head of power in s51 contains an ‘implied
incidental power.’
 Zines says a law will be incidental if
o “directly affects” the subject matter of a HoP, or
o Is ‘conducive’ to it; or
o Is “an appropriate means” or “reasonably necessary” to the effectuation of a
purpose within power
 This incidental power is to legislate in relations to matters necessary to effectuate the
main purpose of the power (Marrickville Margarine)
3. Characterisation of law
what is the subject matter of law? To determine this consider whichh rights, duties,
powers and privileges the law intends to change, regulate or abolish-Kitto J in Fairfax
4. Sufficient connection
A. even if a law cannot be characterised as falling within the HoP, it may be
characterised as incidental if it is reasonably connected to that power
B. This can be used as a conclusion to the scope/characterisation question
5. Conclusion
Consider: reading down/severance/Invalidity
TRADE AND COMMERCE (LECTURE 2) s51(1)

S51(i) the parliament shall, subject to this Constitution, have power to make laws for the POGG
of the Cth with respect to trade and commerce with other countries, and among the states.

o Other HoP can be used to create legislation re trade and commerce (doctrine of multiple
characterisation)
o The High Court has sought to maintain the distinction between trade and commerce ‘with
other countries’ and ‘among the States’, and ‘intrastate’ trade and commerce. (burgess)
o Section 51(i) also enables the Commonwealth both to regulate and participate in trade
and commerce with other countries and among the States.

Grannall v Marrickville Margarine


‘…every legislative power carries with it authority to legislate in relation to acts,
matters and things the control of which is found necessary to effectuate its main
purpose, and thus carries with it power to make laws governing or affecting many
matters that are incidental or ancillary to the subject-matter’ (77).

R v Burgess (1936) 55 CLR 608


FACTS: S4 of the Air Navigation Act 1920 (Cth) authorised the making of regulations ‘for
the control of air navigation in the Cth and the territories’. No HoP that deals specifically
with air navigation. Cth tried to use the trade and commerce HoP to pass the law. (S4 does
not draw a distinction between interstate and intrastate – s51(i) has the implied restriction).
Cth argued that the ‘Cth has a plenary power over inter-State trade and commerce, and a
limited power over intra-State trade and commerce. In order to have effective control over
inter-State traffic, there must be control over intra-State traffic…’ 
o HELD (HC): rejected this submission. It was unanimously held that, although
air navigation ‘in the Territories’ could be regulated under the Territories power
in s122 of the Con’n, air navigation ‘in the Cth’ was a general topic that the Cth
could deal with only in limited respects 
o Latham CJ: ‘The Constitution gives to the Commonwealth Parliament power
over inter-State and foreign trade and commerce and does not give it power over
intra-State trade and commerce ... Although foreign and inter-State trade and
commerce may be closely associated with intra-State trade and commerce, the
court has uniformly held that the distinction drawn by the Constitution must be
fully recognised...’ (629). 
o Dixon J: ‘The inconvenience and difficulty of maintaining the distinction
needs no demonstration. But the legislative power is to make laws with respect to
inter-State commerce, and, under that power, the domestic commerce of a State
can be affected only to the extent necessary to make effectual its exercise in
relation to commerce among the States’ (671). 
 Acknowledges the difficulty of operating two separate legislations but
still rejects the Cth’s submission 
o Evatt and McTiernan JJ: ‘..there may be occasions where parts of intrastate
aviation will be seen to occupy so direct and proximate a relationship to
interstate aviation that the agents and instruments of the former will be drawn
into the ambit of the Federal power..’
Second Airlines Case (1965) 113 CLR 54
This is similar to the statute in the Burgess case: Like the legislation in B, this case, Cth was
seeking to claim authority over inter and intra state public transport operations. These were
more carefully drafted than in Burgess – it was claiming a far more focused power – the
specific circumstances were for the safety, regularity and efficiency of air navigation 
FACTS: regulations made under the Air Navigation Act 1920 (Cth) provided that aircraft
involved in ‘regular public transport operations’ required a Commonwealth licence. In
deciding whether to issue a licence the Director-General was to have regard to ‘the safety,
regularity and efficiency of air navigation and to no other matters.
o HELD: this legislation was held to fall within s51(i) – ie the Cth regulations
were valid 
o Kitto J: Considerations of profit and loss are not enough to enliven the
incidental scope of the power – however, if the intrastate interferes physically
with interstate, then it may bring it within the scope of Cth power. seems to mean
that safety concerns were an example of physical interference (this is regulating
safety).

O’Sullivan v Noarlunga Meat Ltd (1954) 92 CLR 565 - re tracing back along chains of
production*s109
Facts: concerned the effect of a licence issued under regulations which prohibited the export
of meat unless the premises used for slaughter, treatment and storage met certain health
requirements, and were registered.
Fullagar J: I would think it safe to say that the power of the Commonwealth
extended to the supervision and control or all acts or processes which can be
identified as being done or carried out for export. The “slaughter for export” of stock
is such an act or process...
As Joseph and Castan observe, ‘The same result may well not have arisen had the
regulations concerned a production activity which could not be so clearly
differentiated from production for the purposes of intrastate trade’
 Export of goods clearly falls under s51(1).
 however does this power apply at the point of exit/entry? 
 Held: yes, permissible for the Cth to have regard to how the products were
prepared in the first place –s51(1) incidental power can reach back along the chain
of production –> it is not confined to the time or point of export 

Attorney-General (WA) v Australian National Airlines Commission (1976) 138 CLR 492
Stephen J: ‘Does, then, s 51(i), by its grant of legislative power over interstate trade
and commerce, incidentally include a grant of power to legislate for intrastate trade
and commerce when its only relationship to interstate trade and commerce lies in the
fact that the purpose of engagement in such intrastate activity is to conduce to the
efficiency, competiveness and profitability of the interstate activity?’ (508).

‘[In the Second Airlines Case] Kitto J drew a distinction between a law protecting
from the danger of physical interference an activity within power and one which
prevents prejudice to matters merely consequential to such an activity, as for instance
the profitability of interstate commercial air navigation ... In light of the foregoing it is
apparent that the permitted exercise of the power conferred by s 19B ... extends
beyond the ambit of that incidental power which s 51(i) carries with it’ (510-511).
However, by 3:2 the legislation was held to be valid insofar as it relied upon the Territories
power (s 122).
CORPORATIONS POWER s51(20)
S51(xx) the Cth Parliament shall, subject to this Constitution, have power to make laws for
the peace, order and good government of the Cth with respect to foreign corporations, and
trading or financial corporations formed within the limits of the Cth.

Reserved State Powers Doctrine /history of corporations powers

initially interpreted narrowly in light of the ‘reserved State Powers doctrine’:

Huddart, Parker & Co Pty Ltd v Moorehead (1909) 8 CLR 330: (overthrown by concrete
pipes)
 The majority held that s51(xx) should be construed narrowly, but were not able to
agree on an appropriate interpretation 
o Their approach reflected the perceived need to protect ‘the reserved powers of
the States’ which was abandoned in the Engineers Case 
o This is because there was a fear that a broader interpretation might prove
uncontrollable 
Isaacs J dissent in this case – he believed that it should be given a broad/wide
interpretation
o All judges are willing to put a limitation on the corporations power, but we see a
fracturing of how this should be achieved.
o Griffith seeking to impose limits drawn from another head of power – trade
and commerce
o Higgins, seeking to impose a different kind of limit, broader than that of
Griffith
o Isaacs, willing to expand commonwealth power more than the other judges,
still seeking to impose a distinction between the external affairs which fall
within Commonwealth legislative power and the internal affairs of the
corporation that do not

The corporations power was subsequently revived:

Concrete Pipes Case (1971) 124 CLR 468


Gave a clear indication that the Cth could enact trade practices legislation by relying chiefly
on s51(xx).
Barwick CJ:
• Section 51(i) contains no explicit or implicit prohibition and does not reserve the
subject of intrastate trade to the States’
• The laws were valid because they were regulating and controlling the trading
activities of trading corporations and thus within the scope of s 51(xx).

WHICH CORPORATIONS?

Section 51(xx) refers to foreign, trading and financial corporations.


• The current activities test is ‘presumed to cover a wide variety of corporate bodies
such as schools, universities, child care centres, hospitals, charities, non-profit
organisations and various other incorporated State statutory bodies’ (Sarah Joseph)
• The ‘purposes’ test may be relevant for determining if a corporation is a constitutional
corporation if it has not yet engaged in business activities. See Fencott v Muller
• Whether any entity is a corporation of a kind is an issue of substance (QLD rail)

Foreign corporations

New South Wales v Commonwealth (Incorporation Case) (1990) 169 CLR 482

Mason CJ, Brennan, Dawson, Toohey, Gaudron and McHugh JJ: ‘To fall within
the [foreign corporations] limb, a corporation must be ... formed outside the limits of
the Commonwealth’ (498).

Trading Corporations

R v Trade Practices Tribunal; Ex parte St George County Council (1974) 130 CLR 533:
Split in the court about whether a corporation is a constitutional corporation or not
Gibbs (majority):
• employed the purpose test – necessary to determine the true character of the
corporation by way of throwing light on its purpose for formation
• the proper conclusion in my opinion is that the County Council is a
corporation constituted for the purposes of local government to provide an
essential service to the inhabitants of an aggregation of local authority areas,
under conditions thought most likely to prove beneficial to them. It is properly
described as a municipal corporation’
Barwick (dissenting)
• ‘To say that a corporation’s description for relevant purposes will be
determined by its activities does not mean, of course, that a corporation which
to any extent engages in trade is a trading corporation. It is evident that the
legislative power given by s 51(xx) is not a power to legislate with respect to
trading. It is a legislative power with respect to some corporations. But a
corporation whose predominant and characteristic activity is trading whether
in goods or services will, in my opinion, satisfy the description...’

Adamson’s Case (1979) 143 CLR 190


Mason J (majority):
• Activities test was applied, NOT the purpose test.
‘Essentially it [the term trading corporation] is a description or a label given
to a corporation when its trading activities form a sufficiently significant
proportion of its overall activities as to merit its description as a trading
corporation.
Whether the trading activities of a particular corporation are sufficient to
warrant its being characterised as a trading corporation is very much a
question of fact and degree.
Stephen J (dissenting):
• ‘[Club] does what it does to promote football, the income that comes its way
being no doubt eagerly gathered in, but only as a means of better promoting its
predominant purpose, the fostering of football… but only those opportunities
which arise incidentally in the carrying out of those very activities which it
engages in in the furtherance of its principal objects
• purpose test

Financial Corporations

State Superannuation Board of Victoria v Trade Practices Commission (1982) 150 CLR
282
Mason, Murphy and Deane JJ (majority):
• approach to the ascertainment of what constitutes a “financial corporation”
should be the same as its approach to what constitutes a “trading
corporation”, subject to making due allowance for the difference between
“trading” and “financial”...’ (303).
• Activity test: investment activities: ‘The facts demonstrated the appellant
engages in financial activities on a very substantial scale’ (306) (the trading or
financial activity need not therefor be the corporation’s predominant activity
• “Even if we confine our attention to such aspects of the appellant's investment
activities as involve the making of commercial activities and housing loans. its
business in this respect is very substantial and forms a significant part of its
overall activities, No doubt these activities are all entered into for the end
purpose of providing superannuation benefits to contributors, but, as we have
seen, this circumstance constitutes no obstacle to the conclusion that the
appellant is a financial corporation”
Gibbs CJ and Wilson J (Dis)
• Mere quantity is not determinative. the financial activities of a corporation
may be substantial in a quantitative sense and yet no more than incidental and
therefore insignificant in relation to the other activities of the corporation

This was confirmed later in Tasmanian Dam case: the idea that a corporation will be a
trading or financial corporation if its trading and financial activities form sufficiently
substantial portions of its overall activities.

Which activities can be regulated


 Narrow view: The aspects or activities that the Commonwealth can regulate must have
something to do with the characteristic that brings corporations within Commonwealth
power. This would mean, for example, that only the trading activities of trading
corporations can be regulated.
 Broad view: There are no limits at all. Provided that the corporation has the
characteristics that bring it within section 51(xx), any aspect or activity of that
corporation can be regulated by the Commonwealth.
 Tasmanian Dam didn’t resolve this point of law

Fontana
Gibbs CJ: ‘The words of par (xx) suggest that the nature of the corporation to which
the laws relate must be significant as an element in the nature or character of the
laws, if they are to be valid…’ (182).

Mason J: ‘The power should, therefore … be construed as a plenary power with


respect to the subjects mentioned free from the unexpressed qualifications which
have been suggested’ (207-208).
Re Dingjan (1995) 183 CLR 323:
ISSUE: could the Cth Con’n support s127C(1)(b) – legislation that gave the IRC the power to
examine unfair contrcts and set aside contracts or vary them The seven judges accepted that
the critical question must be what degree of relevance or connection to constitutional
corporations is necessary for characterisation as a law ‘with respect to’ those corporations 
HELD (4:3): s127C(1)(b) could not be supported by s51(xx) 
The majority took a narrow approach (s127C(1)(b) could not be supported by
s51(xx)) 
o Dawson J – the fact that it is a trading or financial corporation should be
significant in the way in which the law relates to it.’. Here it is invalid as
appears to regulate contracts which may not be In connection to this
function 
o Toohey J – a law must operate on rights, duties, powers or privileges of
corporation in such a way as to evidence sufficient connection between law and
subject matter. Must be a substantial, not tenuous connection. Not enough to
identify corporations as a vague reference to point to other activities. Here there
was no connection, therefore invalid 
o McHugh J: focus on two step characterisation 
 1. Determine character of the law with reference to rights, power,
liabilities, duties and privileges; 
 2. Is the law so characterised/connected with the HoP? 
 Consider practical and legal operation of law to determine if
connection. Connection is sufficient unless it is so insubstantial, tenuous
or distant. 
 If a law regulates the activities, functions, relationships or business of
s51(xx), no more is needed 
 If law is operating on conduct of outsiders, will not be within s51(xx)
unless that conduct has significance for the trading/financial/foreign
corporation 
o Brennan J: to attract support of s51(xx), it is not enough that law applies to
constitutional corporations and other persons indifferently. It must discriminate
between constitutional corporations and other persons (either by reference to
rights/duties/liabilities or by reference to persons whom affected). S127C(1)(b)
was too wide and did not discriminate 
 Minority (Mason, Gaudron and Deane JJ) – picked up by Workchoices 
o Mason CJ: ‘The power must be construed as a plenary power… the power
extends to the enactment of laws dealing with activities undertaken for the
purposes of the business of constitutional corporation’ 
o Gaudron J: ‘S51(xx) extends to laws prescribing the industrial rights and
obligations of corporations and their employees and the means by which they
are to conduct industrial relations’ 
 Business functions and activities of constitutional corporations and to
their business relationships 
 Extends to persons by and through whom the corporations carry out
those (business) functions and activities and with whom they enter into
those (business) relationships 

Work Choices Case (2006) 


arose from the Howard government’s attempt at reshaping industrial relations in Australia
by the Workplace Relations Amendment (Work Choices) Act 2005 (Cth)
FACTS: Workplace Relations Amendment Act replaced s51(35) with s51(xx) as the basis of
the Act. Action brought by States and 2 unions. Principle object of the Act was to provide a
framework for cooperative workplace relations promoting the Australian economy, removing
state-based relations, encouraging education, improving living standards etc. Meaning of
employer in s16 was limited to that defined in s5 and 6 – ie defined to mean a constitutional
corporation with the meaning of s51(xx) 
Gleeson CJ, Gummow, Hayne, Heydon and Crennan JJ:
 Emphasised that what was ‘discarded in the Engineers’ Case was an approach
to constitutional construction that started in a view of the place to be
accorded to the States formed independently of the text of the Constitution’.
 On assertions regarding the federal balance, said that the ‘point at which the
legislative powers of the Federal Parliament and the legislative powers of the
States are to be divided lest the federal balance be disturbed’ cannot be
clearly identified
 On submissions that the scope of s 51(xx) should be restricted with reference
to s 51(xxxv) observed: ‘Why should HoP, particularly s 51(xx), which are
relied upon by the Commonwealth as supporting a law such as Pt 7, be
construed as not doing so for the reason that s 51(xxxv) identifies particular
means for the prevention and settlement of certain industrial disputes? The
other HoP should not be so construed’
Kirby J (dissenting):
• The corporations power is restrictions placed on laws regarding industrial
disputes by s51(xxxv). What is forbidden is basing a law on one HOP (i.e.
corporations power) when it is clearly a law with respect to another head of
power (i.e. industrial disputes);
• the idea of a fair go that was at the heart of federal workplace laws is
destroyed which has the potential to affect the core values that shaped the
Australian Community and Economy
• the high court should be attentive to the federal character of the Constitution.
Callinan J (dissenting)
• As much as the corporations power may purport to support the legislation, the
power is still subject to the restrictions of the industrial affairs power for industrial
affairs legislation;
Impacts:
- Laws could be constitutional where they controlled the people who represent the
corporation or through the Corporations Act. The corporations speak or act through
persons. Persons external to the corporation could also have their laws captured (lots of
issues for traditional states areas) 
- confirms considerable scope of corporations power following from Gaudron J in
Dingjan. 
- allows Cth power to regulate all aspects of corporations, including entering fields of
traditional state responsibility eg health, education, housing, occupational health and
safety, water, transport etc. 
FISCAL/FINANCIAL POWERS
Taxation power, Appropriation and spending, s96 Grant Power

Background and vertical fiscal imbalance

Vertical Fiscal Imbalance is the difference between the shares of revenue collection and of
expenditure among various tiers of governments (EG federal and states). Horizontal fiscal
equalisation is a response to this.

Fiscal powers and Vertical Fiscal Imbalance (VFI):


 Ch IV – Cth Constitution
o Cth control over customs & excise (eg s 90)
o Free trade within Cth (s 92)
o Payments to States – transitional arrangements with respect to customs & excise
revenue (eg s 87, s 95), distribution of ‘surplus’ (s 94) and grants (s 96)

Case example of vertical fiscal imbalance:

South Australia v Commonwealth (the First Uniform Tax Case) (1942) (later followed by
Victoria v Commonwealth (Second Uniform Tax Case) (1957)
Pre 1942 – both State and Federal income tax levied and States did so inconsistently
from each other.
Package of legislation put in place to increases Cth tax rate, put payment of Cth tax
prior to any state tax and other measures to put Cth tax first.
Cth grant of lost State tax income to States that ceased to levy tax.
HELD: scheme held by majority to be valid (except for reliance on incidental power
for priority of tax payments in Second Uniform Tax case).

Taxation power
S51(ii) ‘The Parliament shall, subject to this Constitution, have power to make laws for the peace,
order, and good government of the Commonwealth with respect to:
…….
(ii) taxation; but so as not to discriminate between States or parts of States; ’

Important provisions

S51(ii): Power to make laws with respect to Taxation but so not to discriminate between
states or parts of states

S81: All taxation must go into the Consolidated revenue fund to then be appropriated for
purposes (read with s83)

Procedural provisions
S53: Senate can’t propose or amend any taxation bills
o May be non-justiciable when dealing with acts not proposed laws -Northern Suburbs
S55: Laws imposing taxation shall deal only with the imposition of taxation, and any
provision therein dealing with any other matter shall be of no effect.
o Air Caledonie: Amendment is invalid if insert tax into non-tax not the prior act
Laws imposing taxation, except laws imposing duties of customs or of excise, shall deal with
one subject of taxation only; but laws imposing duties of customs shall deal with duties of
customs only, and laws imposing duties of excise shall deal with duties of excise only.

(1)Is the financial impost a tax?

Matthews v Chicory Marketing Board (Vic) (1938) 60 CLR 263, 276: What is a tax?: a
compulsory exaction of money by a public authority for public purposes, enforceable by law,
and is not a payment for services rendered’.

Air Caledonie International v Commonwealth While both positive and negative elements
must be taken into account these are not definitive or ‘exhaustive’

IS:
- Compulsory exaction of money
- Enforceable by law
- Exacted by a public authority for public purposes

ISN’T
- A fee for service or licence fee
- A fine or penalty
- Arbitrary

+ Positive elements - Negative elements


- ‘a charge for the acquisition or use of property, a
Compulsory exaction of money fee for a privilege and a fine or penalty imposed
for criminal conduct or breach of statutory
obligation Air Caledonie International at 467) and
-Matthews see s 53
Enforceable by law A tax can’t be a payment for services
rendered nor payment of fee for licence
Matthews INDICIA OF TAX: Air Caledonie International at
467).
-‘If the person required to pay the exaction (fee) is
given no choice about whether or not he acquires
the services and the amount of the exaction has no
discernible relationship with the value of what is
acquired, the circumstances may be such that the
exaction is, at least to the extent that it exceeds that
value, properly to be seen as a tax’ (Air Caledonie
International at 467).

Compare with Airservices:


Exacted by a public authority for public A tax can’t be merely imposing a fine or
purposes pecuniary penalty (s 53) (argue if you can avoid
Australian Tape Manufacturers Association Ltd v the tax in someway)
Commonwealth (1993) 176 CLR 480 Northern Suburbs General Cemetery Reserve
‘royalty’ charged on sale of blank tapes to ‘collecting Trust v Commonwealth (1993) 176 CLR 555
society’ (not a public authority) to pass onto copyright -‘not a penalty since the liability to pay
holders (rather than Consolidated Revenue Fund) still a the impost does not arise from any failure
tax and not essential that paid to a public authority if for
‘public interest’ or ‘public purpose’ (Mason CJ,
to discharge antecedent obligations’ (at
Brennan, Deane and Gaudron JJ and with Dawson, 587 per Dawson J).
Toohey and McHugh JJ dissenting)
o CRF - Presumed public purpose

-Cf Luton v Lessels (2002) 210 CLR 333 Not arbitrary:


FACTS: unpaid child support payments under Child Criteria determining tax liability must not be
Support (Registration and Collection Act) 1988 (Cth) arbitrary – MacCormick - must be possible to point
collected through CRF and passed onto supported child. to the criteria themselves, but it also must be
HELD – not a tax as not raising revenue for Cth and a possible to show that the way in which they are
number of judges cast doubt on ATM. applied does not involve the imposition of liability
o scheme for the creation and adjustment of in an arbitrary or capricious manner
private rights and liabilities is NOT
taxation. [344] 695 -criteria determining tax liability must not be
o every tax that is raised must be paid into the ‘arbitrary. Liability is imposed by reference to
CRF. But the converse is not universally true’ criteria which are sufficiently general in their
(Gaudron and Hayne JJ). application and which mark out the objects and
o Gleeson CJ- at [13]- raising revenue not subject-matter of the tax…Not only must it be
‘universal determinant’ but ‘often… significant possible to point to the criteria themselves, but it
in deciding’ if taxation – ‘That is the most must be possible to show that the way in which
usual form of public purpose involved’. they are applied does not involve the imposition of
o here payment is to facilitate enforcement of liability in an arbitrary or capricious manner…in
‘private rights and liabilities’. relation to that assessment the ordinary processes
of review and appeal are open.’ (MacCormick v
Roy Morgan Research Pty Ltd v Federal Commissioner Commissioner of Taxation (1984) 158 CLR 622,
of Taxation (2011) 244 CLR 97 [49] (JJ) – FACTS 639-42 (Gibbs CJ, Wilson, Deane and Dawson JJ).
(whether superannuation guarantee charge on
employers a tax and resulted in payment from CRF to
employee as superannuation shortfall)
HELD: unanimously held was a tax and was imposed
for public purposes
DECISION
o not necessary to resolve disagreement after
Luton but stated that Australian Tape was
‘large and controversial step’ and depends on
what ‘“non-public” authority’ means (at [37]).
o Imposition into CRF is made for public
purpose, but does not conclusively establish it
as a tax
Note at [19] that ‘public purpose’ ≠ ‘public interest’.
‘submission by the appellant that the Charge is invalid
because the legislation confers upon employees a
“private and direct benefit” cannot be accepted… It is
settled that the imposition of a tax for the benefit of the
CRF is made for public purposes. That is not to say that
the receipt of funds into the CRF conclusively
establishes their character as the proceeds of a tax. But
it does establish in the present case that the Charge is
imposed for “public purposes” and thus, if other
necessary

Exacted by a public authority for public purposes issue


Tape, Luton, Roy Morgan
Broad view in Tape,
Narrower view in Luton and Roy Morgan
Court seems to be moving to a narrower approach

Ask, do we know from the facts whether the money is going into the consolidated revenue fund?

KEY DIFFERENCES BETWEEN ATM AND LUTON:


Tape held that for a tax to be for public purposes it does not have to be paid into the CRF.
Luton held that a scheme for the creation and adjustment of private rights and liabilities is
NOT a taxation. Gaudron and Hayne JJ also in obiter found that every tax must be paid into
the CR fund to be for public purposes but that the converse – that everything paid into the CR
fund is not true (also McHugh J at 361).
Roy Morgan: doesn’t resolve Tape and Luton but holds that ‘it is settled that the imposition
of a tax for the benefit of the CRF is made for public purposes.’

Fee or tax issue

Air Caledonie International v Commonwealth (1988) 165 CLR 462

FACTS: A ‘fee for immigration clearance’ on incoming airline passengers (citizens and non-
citizens) by airlines introduced by s 34A which was inserted into Migration Act 1958 (Cth)
by s 7 of the Migration Amendment Act 1987 (Cth). Airlines then paid fee onto Government
even if passenger had not paid it.

HELD in a unanimous judgment: that fee was a tax –

DECISION
While both positive and negative elements must be taken into account these are not definitive
or ‘exhaustive’
➢ This fee for immigration meets the 3 positive attributes → prima facie attached → compulsory, public
purpose & enforceable
➢ Looking at negative criteria → if this fee were only applicable to non-Australian citizens, it would have
been arguable that it was not a tax because it could be considered a fee for the privilege of entering
Australia.
o But it applies to everyone, indifferently. It does not matter what their citizenship is, payable by
both Australians and non-Australians: therefore it cannot be a fee for a privilege of entry as
Australians are allowed to enter the country without the need for Executive clearance.
o If it was actually just to non-Australians, it wouldn’t have mattered whether it was for services
rendered, it would not have been a tax.
➢ Not a fee for service rendered:
1. Request: Any requirement to submit to immigration procedures cannot be a service that is at the
request. It is being imposed on you instead. No choice about whether or not to acquire the service →
therefore it can’t be considered a ‘service’
- no ‘particular services provided’ because it is being imposed, there is no
choice

Compare with: Canadian Airlines(2000)


FACTS: Concerned unpaid airline operator charges under the Civil Aviation Act 1988 (Cth)
HELD: fee for service and not a tax

DECISION
Fee was not imposed to raise revenue, they were to recover cost of providing services.

Gleeson CJ and Kirby J


‘not all taxation has as its primary purpose the raising of revenue…Even so, the
presence or absence of such an objective will often be significant.
…In this case: the charges were not imposed to raise revenue; the charges were
undoubtedly charges for the provision of services and facilities; the charges were
imposed to recover the cost of providing such services and facilities across the
entire range of users; the charges for categories of services were reasonably related
to the expenses incurred in relation to the matters to which the charges related; the
services and facilities were, of their nature, part of an activity which must be highly
integrated in order to be effective; there was a rational basis for such discrimination
between users as existed. 
In those circumstances, there is no warrant for concluding that the charges amounted
to taxation on the ground that they exceeded the value to particular users of particular
services or the cost of providing particular services to particular users. 
It has not been shown that the subject charges were such as to amount to taxation.’

(2)Is it a law with respect to taxation?: characterisation

A law is characterised as with respect to taxation even if raising revenue is the secondary, not
primary purpose - Northern Suburbs, Fairfax

Northern Suburbs v Commonwealth (1993)


FACTS – Constitutionality of a training guarantee charge challenged – it was levied on
employers who fell short of annual training expenditure requirements.
HELD: a tax and irrelevant that purpose of raising revenue not set out explicitly in Act.

Maso CJ, Deane, Toohey and Gaudron J:


‘the fact that the revenue-raising burden is merely secondary to the attainment of
some other object or objects is not a reason for treating the charge otherwise than
as a tax.

Fairfax v Federal Commissioner of Taxation (1965)


FACTS: 1961 amendment to Act 1936 (Cth) gave income tax exemption to superannuation
funds if the fund invested in ‘public securities including a certain percentage of
Commonwealth securities’.

HELD: was a tax with respect to s 51(ii)


Kitto J:
 it is by no means a settled doctrine that a law which purports to provide for a
tax upon behaviour is in substance not a law with respect to taxation if it
exhibits on its face a purpose of suppressing or discouraging the behaviour and
is to be explained more convincingly as a means to that end than as a means to
provide the Government with revenue’.
 the enactment does not prescribe or forbid conduct. Its character is neither
fully nor fairly described by saying that it makes trustees of superannuation
funds liable to pay for failing to do what the legislature wishes
cf Menzies J:
 There may be laws ostensibly imposing tax which, nevertheless, are not laws
with respect to taxation. For example, a special prohibitive tax upon income
derived from the sale of heroin or from the growing or treatment of poppies…
imposing penalties described as taxes for participation in it.
 The problem in every case is, to ascertain from the terms of the law impugned
its true nature and character
 BUT NB: Menzies J comment here was disavowed in Second Fringe Benefits
Tax Case (1987) 163 CLR 329 at 353-4: Mason CJ, Wilson, Dawson, Toohey
and Gaudron JJ.

(3)Do any special tax limits apply

(a) Discrimination: (must not discriminate between States or parts of States) – s51(ii)
+s99 (Moran, Fortescue

s 51(ii) precludes tax being levied in a discriminatory way between States or parts of States
and s 99 prevents the Cth from ‘giving preference to one State or any part thereof over
another State or any part thereof’ at least ‘by any law or regulation of trade, commerce, or
revenue’

‘discrimination’
Fortescue Metals –
Concerned whether Minerals Resource Rent Tax contravened s 51(ii) or s 99. The taxable
payable was related to level of State royalties payable and these varied nationally.
per Kiefel J: If a Commonwealth taxation law provides that the same measure is to apply to
all persons or things subject to the tax, it would not generally be regarded as likely to
discriminate in fact. Where a difference results from the operation of a taxation law, the
question arises whether that difference is accounted for by the geographical situation of the
subject of the tax. Importantly, for there to be the discrimination of which s 51(ii) speaks, the
difference must be produced by the Commonwealth law itself and by reference to that
geographical situation. There may not be discrimination where the difference results from the
provisions of a State law. Section 51(ii) does not prohibit a taxation law from operating
differentially in all respects. It does not require that a taxation law control the effect of other,
external, factors which may be productive of a difference’.

Moran (1939)
FACTS – Federal tax was levied on flour millers. The Wheat Industry Assistance Act 1938
(Cth) gave grants to the States which were given on condition that the money was given to
growers. Tasmania did not have any wheat growers so condition was that money given to
flour millers.
Latham CJ – no discrimination when identical tax applied to all flour millers.

‘preference’
Crowe v Commonwealth, Elliot v Commonwealth: preference means a ‘tangible advantage’,
only applies (under s 99) if ‘law or regulation discriminates between those states’
(Queensland Nickel)

(b) Property Limits – s114


S114: States may not raise forces or impose any tax on property of any kind belonging to the
Cth nor shall the Commonwealth impose any tax on property of any kind belonging to a State
(c) Procedural Limits
S53: Powers of the House in respect of Legislation
- Senate can’t propose or amend any taxation bills
- A contravention of s53 may not be justiciable – Northern Suburbs
S55: The tacking provision
- ‘Laws imposing taxation shall deal only with the imposition of taxation, and
any provisions therein dealing with any other matter shall be of no effect’
- Amending acts
- Can sever
- Tape
- Air Caledonie
S81 – All revenues or moneys raised or received by the Executive Government of the
Commonwealth shall form one Consolidated Revenue Fund
Tape
Roy Morgan

Appropriation and Spending


S81 – Consolidated Revenue Fund
All revenues or moneys raised or received by the Executive Government of the
Commonwealth shall form one Consolidated Revenue Fund to be appropriated for the
purposes of the Commonwealth in the manner and subject to the charges and liabilities
imposed by this Constitution.

S83 – Money to be appropriated by law


No Money shall be drawn from the Treasury of the Commonwealth except under
appropriation made by law

i) Scope of a Commonwealth Appropriation :‘earmarking’ not spending

Appropriation is simply the ‘earmarking’ of money – the setting aside money for a particular
purpose – Pape
S81 does not create a legislative power to confer on the Executive the power to spend what is
appropriated – Pape

Per Combet and Wilkie, the appropriation of money from the Treasury under s83 may be done
so under a very general purpose – Combet with a degree of specificity determined by the
Parliament – Wilkie.

ii) The scope of the Commonwealth’s Ability to Spend?

S81 and s83 are not the source of the Commonwealth’s power to spend money – Williams

Pape: Spending must fall within the scope of a legislation within the Commonwealths HOP
or by the executive powers under s 61
If spending is not valid - the commonwealth can just use the grant power

Pape
enacted to fund stimulus payments during the global financial crisis
For majority – expenditure supported by legislative nationhood power – s 61 coupled
with s 51(xxxix).
Heydon J: Section 81 does not create a “legislative power” to confer on the Executive
the power to spend what is appropriated.

Williams v Commonwealth (2012) 248 CLR 156


FACTS – constitutional challenge to funding of chaplaincy services in public schools.

HELD – with some exceptions, typically valid Cth legislation required to support
spending of money previously appropriated from Consolidated Revenue Fund.
-spending here invalid by majority as lacked a constitutional basis whether legislative
or constitutional (such as executive action giving effect to statute, prerogative power,
nationhood power or the ‘ordinary and well-recognised functions of Government’).

Williams v Commonwealth (No 2) (2014) 252 CLR 416


FACTS – constitutional challenge to the Financial Management and Accountability Act 1997
(Cth) which sought to accommodate a range of funding initiatives including the chaplaincy
scheme.

HELD by Court unanimously to be invalid as the Act lacked a constitutional basis.


French CJ, Hayne, Kiefel, Bell and Keane JJ: [86] – ‘To hold that the Parliament may
make a law authorising the expenditure of any moneys lawfully appropriated in
accordance with ss 81 and 83, no matter what the purpose of the expenditure may be,
would treat outlay of the moneys as incidental to their ear-marking’.

Grants Power – s96


buzzwords: tax, fee, levy, charge, impost, payment.

S 96 During a period of ten years after the establishment of the Commonwealth and thereafter until
the parliament otherwise provides, the parliament may grant financial assistance to any state on such
terms and conditions as the parliament thinks fit.

(1).Scope of the Power

Through Federal Roads, and First/Second Uniform Tax, the grants power is construed very broadly as
they have the power to any such terms and conditions as the parliament thinks fit.
Dixon J’s narrow view has not prevailed

South Australia v Commonwealth (the First Uniform Tax Case) (1942) 65 CLR 373
HELD: States Grants (Income Tax Reimbursement) Act 1942 (Cth) majority found that
supported by s 96 (Starke J dissenting)
Latham CJ:
a. Can induce: ‘there is no legal compulsion to yield…Action may be brought about
by temptation – by offering a reward – or by compulsion. But temptation is not
compulsion’ (at 417)

b. Can create ‘the need for assistance’ which s 96 is then used to address (at 413)

c. Can’t coerce: ‘But the position is radically different, it is urged, if the so called
inducement practically amounts to coercion…the Commonwealth Parliament
could not pass a law compelling a State to surrender the power to tax incomes or
prohibiting the exercise of that power by a State…a person whose hand is
physically propelled by another person against his will so that it strikes a blow is
not guilty of assault, But it would be no defence to allege that he really could not
help striking the blow because he was offered £1000 for doing it’ (at 417).
d. Can give preferential State treatment: ‘Discrimination in expenditure between
States is found in every Commonwealth budget’ (at 429)

Victoria v Commonwealth (Second Uniform Tax Case) (1957) 99 CLR 575 –Uniform tax
arrangement ceased to be temporary
States Grants (Tax Reimbursement) Act 1946-1948 (Cth) – valid
Dixon CJ – emphasised that s 96 is a power ‘to grant money’ and ‘may be taken to
fall short of authorising a special attempt to control the exercise of the constitutional
powers of the States’ (at 610)
-can impose terms and conditions on grant of funds and ‘money may supply the
inducement to comply with the term or condition’ (at 610).

(2).Constitutional limits on a s96 grant

(a) States can’t be legally forced to accept grant on conditions imposed, but economic
inducement is permissible
Second Uniform Tax Case
Dixon CJ at 609: ‘By coercive law is meant one that demands obedience
(b) Can’t impose unconstitutional or illegal conditions on States (First and Second Uniform Tax
Case) (eg you can’t violate a constitutional provision for example section 92, or s90)

(c) Can’t be used to evade limits on Cth power (such as s 116 Religion and in s 51 just
compensation) - ICM Agriculture. The states can still do so, but they cant be required to
do so - (Commonwealth could advise the States to acquire property not on just terms but cant
require it)

ICM Agriculture v Commonwealth (2009) 240 CLR 140

FACTS - related to intergovernmental water funding under the National Water Initiative
and a NSW project which used s 96 to provide a grant with conditions relating to bore
usage. Plaintiffs claimed this breached the requirement that property only be acquired on
just terms (s 51(xxxi))

HELD – s 51(xxxi) not breached as no property acquired.


French CJ, Gummow and Crennan JJ – at 170 – ‘the legislative power of the
Commonwealth conferred by s 96 and s 51(xxxvi) does not extend to the grant of
financial assistance to a State on terms and conditions requiring the State to acquire
property on other than just terms’.
- Found that you can’t use grants to circumvent constitutional restrictions – you can’t
use the grants power – s96 to get the states to ‘do the cth’s dirty work’ [170] per
French CJ, Gummow and Crennan JJ

(d) Cant be a colourable device - Moran

Grants cannot be made under s96 under the pretence of financially assisting a State but with
the real objective or purpose of effectuating a discrimination in regards to taxation – BUT
difficult to prove this

(e) Can discriminate between States and be to ‘any State’ – Moran

Section 96 is a means of evening such inequalities in accordance with the judgement of


Parliament. That section is not limited by any prohibition on discrimination’
Tax practice problem question 1

The Commonwealth Treasurer, Sally Gifford, is concerned that the Commonwealth’s tax
revenue is likely to be significantly reduced due to COVID-19, leaving insufficient funds to
tackle the growing threat posed by climate change.
 
Sally introduces into the House of Representatives the Once in a Generation Change Climate
Fund Bill 2021 (Cth) (‘the Bill’). If enacted, it would compel a person worth more than $1
billion to pay a wealth tax at an incontestable minimum rate of 8% on all wealth above $1
billion to a ‘Once in Generation Climate Fund’ by 31 December each year. A billionaire
could elect to pay the tax at a higher rate depending on the contribution that they wish to
make to the Once in a Generation Climate Fund. It is estimated that at least 150 billionaires
would be subject to the Bill (with most residing in WA, NSW and Victoria) and their
contributions would be recognised by each receiving, and having installed by the
Commonwealth, a solid gold ‘Once in a Generation Climate Fund’ plaque, the size of which
will vary based on the tax paid.
 
Samara Safford is a tech billionaire who seeks your advice about the constitutionality of the
Bill if enacted.

Question 2
Assume the Commonwealth Parliament enacts:
• Fees and Charges (Small Shops) Act 2022 (Cth)
• Appropriation Act 2022 (Cth)
• State Grants (Assistance to Small Shops) Act 2022 (Cth)
 
Also assume that this Commonwealth legislation has been implemented to carry out one
aspect of the new Commonwealth Government’s “equality” policy. To put small shop
owners in the cities and metropolitan suburbs on an equal basis with farmers in non-
metropolitan country areas who during 2020-1 received Commonwealth Government
financial assistance in the form of drought relief, the above Commonwealth legislation
includes the following provisions:

Section 2 of the Fees and Charges (Small Shops) Act 2022 (Cth) requires that farmers must
pay a prescribed fee of $120,000 to the Commonwealth Department of Primary Industry to
obtain Commonwealth Government information and non-financial assistance relating to
farming unless a farmer shops at and spends $80,000 per annum at small city or metropolitan
shops.
 
Section 3 of the Appropriation Act 2022 (Cth) provides that the Commonwealth Treasurer
may issue out of the Commonwealth CRF up to $500m for the purpose of providing national
partnership grants to States including for disaster relief.
 
Section 4 of the State Grants (Assistance to Small Shops) Act 2022 (Cth) provides that the
Commonwealth Minister for Trade may give to one or more States up to a maximum of $50m
to pay off small shop loans and develop new small shop enterprises on condition that the
State agrees to cease collecting State taxes from small shop owners.
 
Susan Smith, a wheat farmer, in Kalgoorlie has received more than $750,000 in
Commonwealth drought relief payments and seeks your constitutional law advice.

Section 2
Introduction
The advice that follows is for Susan Smith who wants Section 2 of the Small Shops Act 2022
to be found invalid so as to not pay the 120,000. The relevant head of power for the act is
s51(ii) which gives the Commonwealth Power to legislate with respect to taxation.

Is the financial impost a tax?


A tax is a compulsory exaction of money by a public authority for public purposes,
enforceable by law, and is not a payment for services rendered’ - Latham CJ in Matthews.
These criteria must be considered however they are not exhaustive - Air Caledonie.

- We don’t really know if its going into the CRF. Talk about implications of if it
doesn’t or does go.
- Benefiting public at large not merely enforcing private rights – Luton
- It is a fee for services?: probably not
- Is it a penalty

Is it a law with respect to taxation

A law is characterised as with respect to taxation even if raising revenue is the secondary, not primary
purpose - Northern Suburbs. The law issues a tax and because of the doctrine of dual characterisation
– Fairfax, the law is with respect to taxation.

Do any Tax limits apply

S81 – is it going into CRF

Do any other limits apply


Immunities, inconsistency, IFPC

Conclusion

Then do Section 3 (appropriation)


Per Combet and Wilkie, the appropriation of money from the Treasury under s83 may be
done so under a very general purpose – Combet with a degree of specificity determined by
the Parliament – Wilkie.

Section 4

Section 96 empowers the Cth to grant financial assistance to any state on such conditions they see fit.
This grant [description of grant]. The issue is whether this is a valid exercise of s96.
Then look at limitations.
Difficult to challenge if you see this as a grant of assistance as opposed to controlling state power.

ICM Agriculture (2009) (LIMITS ON GRANTS: EVADING CTH POWER)


Commonwealth and the States get together and protect water resources in Australia. Concern that overusing
water things, they need to prevent environmental damage and reduce water usage. The biggest user of water =
agriculture. The existing scheme, if you are an agriculture user of water, you apply to the State government to
extract water from the ground. Gives you a certain entitlement per year. They agree to reduce everyone’s water
allocation. They were going to have a reduction in what you could take out. There was commonwealth funding
for this per s96. As a result of this, ICM said that you have acquired our property (licence) and acquired without
providing compensation. At the State level, there is no constitutional prohibition in taking property without
compensation. You can take what you want without paying (not constitutionally entrenched). By contrast, there
is a prohibition at the Commonwealth level, need to offer ‘just terms’ if they take property.
note* Magennis and Pye are background to this case.

Arguments from ICM: You’re using the s96 grants to circumvent that restriction. The Commonwealth is not
acquiring the property, it is paying the States to acquire the property as they do not have that acquisition
requirement to provide ‘just terms.’
French CJ, Gummow and Crennan JJ
➢ Commonwealth cannot grant money to a State with a term and condition requiring that State to acquire
property not on just terms. S51(31) doesn’t apply directly to the States, but you can’t use the States to do
the Commonwealth’s dirty work [170] 779.
o ‘The construction involves looking beyond matters of legal form and to the practical effect
of the law in question' about the relationship between s51(31) and s96.
o Refers to the term ‘circuitous device.’ → The banks and shareholders were stripped of
possession and control of their entire undertaking without compliance with s51(31) → in an
informal way. It was done without compliance with s51(31). Dixon J in Magennis used the
expression said it cannot be used.
o S51(31) is a ‘very great constitutional safeguard’…s51(31) ‘has assumed the status of a
constitutional guarantee of just terms…and is to be given the liberal construction appropriate
to such a constitutional provision.’
➢ There seem to be exceptions however. In particular, they talk about the Pye v Renshaw exception.
➢ The Court seems sceptical of this to some extent.
➢ Cast some doubt on whether it’s possible to get around these restrictions by inserting the TCs in an
informal agreement.
➢ Grant that has no Terms and Conditions but there is a side letter (you must do this). An informal
agreement, to get around it, may not reflect a correct understanding of s96 and its relationship with s51 of
the Constitution. [168] 778
➢ The State acquiring the property made reference to the Grant. It was held to be invalid. If there is a clear
link between the law acquiring property and the grant → using the Grant’s power to get around the
constitutional limit in s51(31) and it is therefore invalid.
➢ The informal agreement recouples them in a way.
➢ Magennis should not be opened up or overturned.
➢ S96 and s51(31) should be read together. Magennis is a persuasive analogy (respecting the relationship
between s96 and s116 → freedom of religion)

SUMMARY:
➢ The Cth has a broad power to impose TCs on grants. They can impose TCs that require the States to
pay the money to a non-government entity and can impose TCs that do not relate to a head of power
(DOGS case).
➢ Commonwealth Parliament cannot issue a grant that violates the limitations and restrictions contained
elsewhere in the Constitution. S51(31) and freedom of religion prohibition (Moran and ICM)
➢ Note that the Court has sometimes taken a formal or narrow approach. They have not adopted a very
practically oriented view of the issue at all times. Particularly they have allowed a connection between s96
grant and State law that might be violating the prohibition to be severed. Though there is nothing in the
State law saying that it is acting pursuant to the Cth grant (decoupled), Pye suggests you can use it to get
around it. It was not overruled in ICM. But must say that the Court has expressed some doubts about this.
The Court has certainly questioned the ability to use circuitous devices (informal agreements). Uncertainty
→ Either (Arlen approach)
➢ It is very difficult to demonstrate that s96 grant is violating probation in s51(2) as s96 does allow
discrimination between States and it seems like a broad allowance. (2nd uniform Tax case and
Moran’s) Broad discrimination is permissible under s96, even though formally, Moran says you can't use
it to violate s51(2). It reality, it is impossible to think of a situation where it would actually be violated.
IMPLIED FREEDOM OF POLITICAL
COMMUNICATION 
The implied freedom of political communication is
- implied instead of express; 
- restricted to political communication instead of expression generally; and 
- operates as a limitation instead of a right vested in individuals.

When does it operate?


 Both State and Cth legislation must conform – will be read down or invalidated –
Unions NSW
 Common law must also conform Lange 565
 Freedom extends to all levels of government (local, state and Federal) at any point in
the election cycle Lange 571

Cases
 Lange involved the scope of the common law doctrine of defamation
 Coleman involved a provision that prevented the using of insulting words in a public
place. Ruled an illegitimate burden and thus invalid
 Monis involved a provision that prevented the use of the postal service to harass or
offend. Valid HCA 3:3 split Thus NSWCA decision applied. (Not a strong maj – note
this in hypos)
 McCloy Legislation imposed caps on state political donations and prohibited
donations by certain groups (mainly property developers) Was held valid.

Background to Implied Freedom of Political Communication 


- Murphy J was a strong advocate for such introduction: Attempted to read in a de facto bill
of rights 
- Murphy J was unable to persuade other members of the Board 
- Mason J’s rejection of legalism: should interpret constitution with regards to its ‘values’.

The two leading cases that established that the Constitution embodies an implied freedom of
political communication:
- Nationwide News Pty Ltd v Wills (1992) 
- Australian Capital Television Pty Ltd v Commonwealth (1992) 

Nationwide News Pty Ltd v Wills (1992) 

Facts:The publisher had been prosecuted under the Industrial Relations Act 1988 (Cth) which
provided that: ‘A person shall not ... by writing or speech use words calculated ... to bring a
member of the [Industrial Relations] Commission or the Commission into disrepute.’ The
prosecution related to an article published in The Australian which spoke of the right to work
as ‘regulated by a mass of official controls, imposed by a vast bureaucracy in the ministry of
labour and enforced by a corrupt and compliant “judiciary” in the official Soviet-style
Arbitration Commission.’ 
HELD (unanimous): the section was invalid 
Deane and Toohey JJ 
Since the Engineers, the approach of constitutional interpretation starts with looking
at the text, this case was different. There was a rejection of the textualism of the
Engineers case – looking at the principles underlying the constitution. There are
shades of Dworkinian interpretation n Dean and Toohey’s Judgement. Why? Because
Dworkin is saying you must look at the moral principles underneath the law to
understand its basis.

o Three main general doctrines of government underlie the Constitution:


federalism, the separation of powers, and representative government. 
 Doctrine of representative government gives rise to the implied
freedom of political communication (PARAPHRASED FROM 69)
 ‘The doctrine presupposes an ability of represented and representatives to
communicate information, needs, views, explanations and advice. It also
presupposes an ability of the people of the Commonwealth as a whole to
communicate, among themselves, information and opinions about matters
relevant to the exercise and discharge of governmental powers and functions
on their behalf’ (71-72).
o The implied freedom of political communication ‘operates at two levels.
The first is the level of communication and discussion between the represented
and their representatives ... The second level at which the implication of freedom
of communication and discussion operates is the level of communication
between the people of the Commonwealth’ (74). 
 People of the Cth – if we’re going to form views on political issues in
order to vote for example, we need to be able to have freedom of political
communication with other people in society 
 What about the issues between States and the Cth. Eg, issues in WA
which other states/the Cth may not be interested in 
o ‘...the relationship and interaction between the different levels of government
are such that an implication of freedom of communication about matters relating
to the government of the Commonwealth would be unrealistically confined if
applied only to communications in relation to Commonwealth governmental
institutions ... It is, however, unnecessary to express a firm view in relation to
that question...’ (75). 
 An issue of the state may eventually become an issue of the national as
a whole eg issues of a political party at a state level may affect the views
of that party at a federal level 

Australian Capital Television Pty Ltd v Commonwealth (1992) 

Facts
 Involved a challenge to the validity of the Political Broadcasts and Political
Disclosures Act 1991 (Cth), Pt IIID of which imposed a blanket prohibition on
political advertisements on radio or television during federal election periods (ie
during this period, there could be no political advertising by the political parties) 
 The Act also established a scheme of ‘free time’ for political advertising, 90% of
which was reserved for parties represented in the previous Parliament and fielding a
prescribed number of candidates. 
o Beyond that, political advertising was prohibited 
oRationale for the legislation – issue of spending, it shouldn’t be the case where
political parties with a big financial background could spend more than others;
another aspect was that if there was pressure to raise money, this becomes a
necessity and therefore it could lead to corruption 
 ISSUE: whether Pt IIID was invalid because it infringed a constitutionally guaranteed
freedom of political discussion. 
 HELD: wholly invalid on that basis (Mason CJ, Deane, Toohey and Gaudron JJ) 

Judgement

 Judgment of Mason CJ who was at the forefront of reading these implications in to


the Con’n 
o SAID Engineers should not be taken that implications can never be read into
the Constitution 
o ‘It may not be right to say that no implication will be made unless it is
necessary. In cases where the implication is sought to be derived from the
actual terms of the Constitution it may be sufficient that the relevant intention
is manifested according to the accepted principles of interpretation. However,
where the implication is structural rather than textual it is no doubt correct to
say that the term sought to be implied must be logically or practically
necessary for the preservation of the integrity of that structure’ (135).
o Distinguishing between the text and the structure of the Constitution 
o Regards this case one in which, the implied freedom as part of the
structural commitment to representative government. 
o Freedom of communication as an indispensable element in representative
government
‘Indispensable to that accountability and that responsibility is freedom of
communication, at least in relation to public affairs and political discussion’ (138).
o Infringement: What test to be applied- PROPORTIONALITY
‘If the restriction imposes a burden on free communication that is disproportionate to
the attainment of the competing public interest, then the existence of the
disproportionate burden indicates that the purpose and effect of the restriction is in
fact to impair freedom of communication’ (143-144).
o Is Part IIID valid?
‘As I pointed out earlier, Pt IIID severely restricts freedom of communication in
relation to the political process, particularly the electoral process, in such a way as to
discriminate against potential participants in that process. The sweeping prohibitions
against broadcasting directly exclude potential participants in the electoral process
from access to an extremely important mode of communication with the electorate’
(145)
 McHugh dissent –says s7. 24 guarantee implied freedom
If the freedom is derived from the text and structure of the Constitution alone, this would
enable the freedom to be narrowly construed as being primarily a freedom of discussion
as regards electoral matters, as sections 7 and 24 focus upon the electoral process.
“the people have a constitutional right to convey and receive opinions, arguments and
information concerning matter intended or likely to affect voting in an election for the
Senate or the House of Representatives”
Implied Freedom of Political Communication and Defamation

- The law of defamation is capable of burdening free political


communication
- To what extent can a limitation on legislative power affect rights and
duties in common law?
- Assuming that the implied freedom has application to the law of defamation, should
the courts develop the existing common law or formulate new ‘constitutional
defences’? 
- Theophanous v Herald is one of the cases that considers the issue of
defamation of IFoPC

Theophanous v Herald & Weekly Times Ltd (1994) 

Concerned a member of the Commonwealth Parliament and the former chairman of its Joint
Parliamentary Committee on Migration, Dr Andrew Theophanous, sued for defamation
following the publication in a newspaper of a letter to the editor by Bruce Ruxton. 
The letter attacked Theophanous’ immigration policies and accused him of bias arising from
his ethnic background. Eg reputation 
In response, implied freedom of political communication was invoked 

Joint Judgment of Mason CJ, Toohey and Gaudron JJ 


- ‘If the Constitution ... is at variance with a doctrine of the common law, the latter
must yield to the former’ (126). 
- formulated a constitutional defence of ‘reasonable publication’ with regards to
defamation (available to a person sued for defamation as a consequence of political
discussion about a candidate for office)
- Held that such political discussion cannot be attacked by way of a defamation action
where the publisher of the speech can establish that:
[1] it was unaware of the falsity of the material published;
[2] it did not publish the material recklessly, that is, not caring whether the material
was true or false; and
[3] the publication was reasonable in the circumstances.

McHugh J (dissenting)
- ‘...it seems to me that those judgments in Australian Capital Television and
Nationwide News Pty Ltd v Wills that hold that the institution of representative
government is part of the Constitution independently of the terms of certain sections
of the Constitution unintentionally depart from the method of constitutional
interpretation that has existed in this country since the time of the Engineer’s Case’
(202).

In Lange, the High Court unanimously affirmed the existence of the implied freedom of
political communication but tied its rationale more narrowly to ss 7 and 24 of the
Commonwealth Constitution.

Lange v Australian Broadcasting Corporation (1997) 


David Lange, a former Prime Minister of New Zealand, sued the Australian Broadcasting
Corporation for alleged defamatory statements in a Four Corners television program. 

Unanimously upheld the existence of the implied freedom of political communication

Established the Lange Test – a 2 part test


- ‘First, does the law effectively burden freedom of communication about government
or political matters either in its terms, operation or effect?’ (567). 
- ‘Second, if the law effectively burdens that freedom, is the law reasonably appropriate
and adapted to serve a legitimate end the fulfilment of which is compatible with the
maintenance of the constitutionally prescribed system of representative and
responsible government…?’
IE
1. If there is a law being challenged, does it burden political communication? 
2. If yes, is the law reasonably appropriate and adapted to serve a reasonable end?
(relationship between the law and the objective) 
Implied Freedom of Political Communication post Lange – Four Main
Areas 
- What constitutes political communication?
- To what extent are offensive communications protected by the implied freedom of
political communication?
- How has the proportionality test developed post-Lange?
- The implied freedom of political communication and protests

Does the Implied Freedom of Political Communication apply to State,


Territory and Local Laws?

It is now well-established that the implied freedom of political communication applies to


State and Territory laws. (Unions NSW v New South Wales (2013) 252 CLR 530)
French CJ, Hayne, Crennan, Kiefel and Bell JJ: ‘The complex interrelationship
between levels of government, issues common to State and federal government and
the levels at which political parties operate necessitate that a wide view be taken of
the operation of the freedom of political communication’ [25].

Also, in Attorney-General (SA) v Corporation of the City of Adelaide (2013) 249 CLR 1, the
High Court considered the compatibility of a municipal by-law with the implied freedom of
political communication.

What Constitutes Political Communication?

Theophanous v Herald & Weekly Times Ltd (1994) 182 CLR 104

Mason CJ, Toohey and Gaudron JJ: ‘Indeed, in our view, the concept is not
exhausted by political publications and addresses which are calculated to influence
choices. Barendt states that ((15) Freedom of Speech, (1985) at 152): ‘Political speech
refers to all speech relevant to the development of political opinion on the whole
range of issues which an intelligent citizen should think about’ (123).

Brown v Classification Review Board (Rabelais Case) (1998) 154 ALR 67


‘The Art of Shoplifting.’ It involved a step by step guide to shoplifting.
HELD: not political communication as it’s true character did not concern political or
government matters 
Sundberg J: Although its opening “redistribution” paragraph is “political” in one
sense of the word, its true character is not political because it is overwhelmingly a
manual about how successfully to steal’ (98).
Heerey J: The advocacy of law-breaking falls outside this protection and is
antithetical to it’ (87).

Clubb v Edwards; Preston v Avery (2019) 267 CLR 171

 Kiefel CJ, Bell and Keane JJ: ‘A discussion between individuals of the moral or
ethical choices to be made by a particular individual is not to be equated with
discussion of the political choices to be made by the people of the Commonwealth as
the sovereign political authority’ [29]. MORAL DISCUSSION IS NOT
NECESSARILY THE SAME AS POLITICAL COMMUNICATION

Nettle J: ‘...although abortion is a subject matter of political controversy, it does not


follow that all communications about abortion are political’ [249].

Examples of communication considered political

 Anything related to parliament/politics/representative judgement such as:


(Theophanous) 
 conduct, policies or fitness for office of Govt. 
 discussion about political parties, public bodies, public offices and those seeking
public office 
 the political views and public conduct of persons who are engaged in activities which
has become the subject of political debate (Union/aboriginal leaders,
political/economic comment) 
 Repeal of law/conduct of public officers (Brown) 
 Eg calling someone a ‘corrupt officer’ (Coleman) 
 Immigration advice (Cunliffe) 
 Voting/act of voting (Nationwide News) 
 Judiciary/justice 
Eg communications to act or omissions by the legislature or executive arm and those
communications are relevant to the administration of justice; McHugh J in APLA 

Implied Freedom of Political Communication and Offensive


Communication
To what extent are offensive communications protected by the implied freedom of political
communication?

Coleman v Power (2004) 220 CLR 1 (the Lange test developed)

 Patrick Coleman distributed a pamphlet stating: ‘GET TO KNOW YOUR LOCAL


CORRUPT TYPE COPS’ and ‘KISS MY ARSE YOU SLIMY LYING BASTARDS.’ 
 Police officer approached and asked if he could have one. Subsequent to an altercation
with Constable Brendan Power, Coleman was charged with using ‘threatening, abusive or
insulting words’ (s 7(1)(c)) and distributing printed material containing such words (s
7(1)(d)), under the Vagrants, Gaming and Other Offences Act 1931 (Qld). 
 He was convicted under s7(1)(d) but challenged it as being contrary to the implied
freedom of political communication 

The statute was upheld by the majority, but subject to a particular statutory interpretation.
The words in the statute should be read as referring to ‘provocative speech’.
Majority:
Gummow and Hayne JJ:
 in order for the statute to apply, it’s not enough if the words are merely insulting
or offensive, the point is at which language becomes provocative, which carries a
very specific meaning according to the majority judgment. That is intended to
provoke, or likely to provoke unlawful physical retaliation.
 When looking at the impugned law, it needs to be read down in such a way so that
it is not regulating the civility of discourse. 
Kirby (also majority):
 ‘One might wish for more rationality, less superficiality, diminished invective and
increased logic and persuasion in political discourse ... From its earliest history,
Australian politics has regularly included insult and emotion, calumny and
invective, in its armoury of persuasion. They are part and parcel of the struggle of
ideas’. Offensive communication part of the political process in Australia
Gleeson CJ (dissenting):
 ‘It is open to Parliament to form the view that threatening, abusive or insulting
speech and behaviour may in some circumstances constitute a serious interference
with public order, even where there is no intention, and no realistic possibility,
that the person threatened, abused or insulted ... might respond in such a manner
that a breach of the peace will occu’.up to the parliament to determine what
speech may be curtailed

Monis v The Queen (2013) 249 CLR 92

Facts
 Criminal Code (Cth) Section 471.12 A person is guilty of an offence if: 
(a) the person uses a postal or similar service; and 
(b) the person does so in a way ... that reasonable persons would regard as being, in all the
circumstances, menacing, harassing or offensive. 
o Monis had been writing letters to relatives of soldiers who had been killed in Iraq and the
form of the letters were insulting about soldiers who had been killed (there was also
political content to these letters). He was charged. 
o The accused sought to have their indictments quashed on the basis that s 471.12 infringed
the implied freedom of political communication. 
o He was convicted under the statute. He appealed in light of Coleman. He argued that the
section was invalid and breached implied freedom of political communication 

3v3 equal split

French CJ, Hayne and Heydon JJ found the provision was not valid

Hayne J: ‘...s 471.12 is not directed to a legitimate object or end. The elimination of
communications giving offence, even serious offence, without more is not a legitimate object
or end ... Giving and taking offence are inevitable consequences of political debate and
discourse’.
French and Hayne = draws on Coleman – said that yes the letters were insulting but that is
political discourse (so deal with it) 
Heydon = he didn’t want to allow the legislation and wanted to convict Monis but allowed it
because he had to begrudgingly accept the implied freedom of communication. 

Crennan, Kiefel and Bell JJ found the provision was valid


 ‘Section 471.12 seeks to deter a particular use of the postal service. It may be
taken to recognise a citizen’s desire to be free ... from the intrusion into their
private domain of unsolicited material which is seriously offensive’ [320].
o The section had a specific purpose of protecting privacy
o Unsolicited intrusions into the home present a different kind of issue and
we should recognise an exception to the general proposition in Coleman v
Power
 ‘This does not suggest an effect upon the freedom which could be regarded as
extensive. It does not prevent communications of a political nature which do not
convey such offensive matter ... Section 471.12 does not impermissibly burden the
implied freedom. The Lange test is satisfied.

Clubb v Edwards; Preston v Avery (2019) 267 CLR 171


Both appeals were unanimously dismissed
Neither Act found to be in breach of the constitutional freedom of political communication
(per Kiefel CJ, Bell, Keane)
Concurring (Nettle J) (Gageler J) (Gordon J)(Edelman J)

Kiefel CJ, Bell and Keane JJ: ‘The object of the prohibition is to protect the safety
and wellbeing, physical and emotional, of persons accessing and leaving abortion
clinics and to ensure that women may have unimpeded access to, and doctors may
provide, terminations’ [122].

Gageler J: ‘...the better explanation of [Coleman and Monis] is that protecting against
unwanted or offensive communications is a permissible purpose the capacity of which
to justify a burden on freedom of political communication can vary in different
contexts’ [196].
IN SUMMARY

 As a general rule, legislatures cannot pursue the ‘civility of discourse’ or prohibit


communications that are merely offensive (Coleman v Power (2004) 220 CLR 1).
 However, post-Coleman v Power the High Court has increasingly recognised that
legislatures may prohibit communications that exceed mere offensiveness but fall
short of provocation, especially in the context of communications that engage the
values of privacy and dignity (Monis v The Queen (2013) 249 CLR 92, Clubb v
Edwards; Preston v Avery (2019) 267 CLR 171).

The Adoption of Structured Proportionality

McCloy v New South Wales (2015) 257 CLR 178

Facts
About political donations – the Election Funding, Expenditures and Disclosures Act 1981
(NSW) imposed a cap on political donations, prohibited indirect campaign contributions, and
prohibited the making of political donations by property developers (due to potential
corruption).
M was a property developer in NSW and was unhappy and argued that this breached his
political communication.
McCloy argued that political donations are a form of political communication. 
HELD: the law did not burden the implied freedom of political communication

French CJ, Kiefel, Bell and Keane JJ developed the Lange test as follows:

1. Does the law effectively burden the freedom in its terms, operation or effect?
2. Are the purpose of the law and the means adopted to achieve that purpose
legitimate, in the sense that they are compatible with the maintenance of the
constitutionally prescribed system of representative government’ (compatibility
testing)?
3. Is the law reasonably appropriate and adapted to achieve that legitimate purpose
(proportionality testing)?

Regarding step (1), ‘even a “little” burden on the implied freedom qualifies as an “effective”
burden for the purpose of the first limb of the Lange test’ (Monis v The Queen (2013) 249
CLR 92 at [173] per Hayne J)

Regarding step (2), a ‘legitimate purpose is one which is compatible with the system of
representative government provided for the by the Constitution...’ (McCloy v New South
Wales (2015) 257 CLR 178 at [31] per French CJ, Kiefel, Bell and Keane JJ).

Regarding step (3), in McCloy v New South Wales (2015) 257 CLR 178 at [2] French CJ,
Kiefel, Bell and Keane JJ held that proportionality entails a further three steps:

1. Suitability: Does the law have a rational connection to the purpose of the
provision?
2. Necessity: Is there an obvious and compelling alternative, reasonably practicable
means of achieving the same purpose which has a less restrictive effect on the
freedom?
3. Balancing: The Court must make a value judgment, consistent with the limits of
the judicial function, describing the balance between the importance of the
purpose served by the restrictive measure and the extent of the restriction that it
imposes on the freedom.

Not all members of the Court welcomed the adoption of structured proportionality.

Gageler J preferred a US influenced ‘categorical’ approach whereby the intensity of


review is ‘calibrated to the degree of risk to the system of representative and
responsible government’ [150].

Gordon J preferred the established ‘appropriate and adapted’ test: ‘The questions
stated for the opinion of the Court in this case are able to be answered by reference
to the known questions and tools’ [311].

In Clubb/Preston, five members of the Court – Kiefel CJ, Keane and Bell JJ, Nettle J and
Edelman J – endorsed structured proportionality.

However, Gageler J and Gordon J have continued to dissent on this issue e.g. Comcare v
Banerji (2019) 267 CLR 373 at [97] (Gageler J) and [161] (Gordon J).

In LibertyWorks Inc v Commonwealth of Australia [2021] HCA 18 Kiefel CJ, Keane and
Gleeson JJ, Edelman J and Steward J applied structured proportionality. Gageler J and
Gordon J declined to apply structured proportionality.

Steward J also doubted the existence of the implied freedom:

‘…it is arguable that the implied freedom does not exist. It may not be sufficiently supported
by the text, structure and context of the Constitution and, because of the continued division
within this Court about the application of the doctrine of structured proportionality, it is still
not yet settled law. The division within the Court over so important an issue may justify a
reconsideration of the implication itself’ [249].

Proportionality and Protests

Brown v Tasmania (2017) 261 CLR 328

Kiefel CJ, Bell and Keane JJ adjusted the Lange test as follows:

1. Does the law effectively burden the freedom in its terms, operation or effect?
2. Is the purpose of the law legitimate, in the sense that it is compatible with the
maintenance of the constitutionally prescribed system of representative and
responsible government?
3. Is the law reasonably appropriate and adapted to advance that legitimate object
in a manner that is compatible with the maintenance of the constitutionally
prescribed system of representative and responsible government? (Contains
further 3 steps)

Step 1: Does the legislation burden the IFPC

‘In the present case the answer is clear. Protestors will be deterred from voicing
their protests with respect to forest operations. The freedom is burdened’ [95].

Step 2: Is the purpose of the law legitimate?

The purpose of the Act is to protect ‘businesses and their operations, here forest
operations, from damage and disruption from protestors who are engaged in
particular kinds of protests’ [101]. This purpose is legitimate [102].

Step 3: Is the law reasonably appropriate and adapted?

Suitability

• Section 8(1)(b) fails the test of suitability: ‘It deters a person being in any
business access area on pain of arrest or penalty, even though they may not
present any threat of damage or disruption...’ [135].
• Sections 11(7) and (8) fail the test of suitability: they ‘effect a blanket exclusion of
a whole group of persons from an area by a single direction of a police officer,
even when the police officer could not conceivably have formed a view about
whether each person is about to contravene the Protestors Act’ [136].
Provision was found not to be rationally connected to the legislative aim, thus not
constitutionally valid.
Necessity

• The Forest Management Act 2013 (Tas) demonstrated that the impugned
provisions were not necessary.
• ‘...the premise of the FMA is that persons are able to access forest areas unless
the Forest Manager exercises its powers to exclude them. The Forest Manager
may only exercise those powers in order to perform its functions effectively or
efficiently, or in the interests of safety’ [110].
• ‘The Protestors Act operates more widely than its purpose requires. It is
principally directed to preventing protestors being present within ill-defined
areas...’ [140]

Clubb v Edwards; Preston v Avery (2019) 267 CLR 171

Kiefel CJ, Bell and Keane JJ

Step 1: Does the legislation burden the IFPC

The Victorian Act and the Tasmanian Act effectively burden the implied freedom.
Step 2: Is the purpose of the law legitimate?

The Victorian Act and the Tasmanian Act have the legitimate purposes of protecting
the safety, well-being, privacy and dignity of persons accessing abortion clinics.

Step (3)

Suitability

The Victorian Act and the Tasmanian Act are suitable in that they are rationally
connected to the statutory purposes of protecting the safety, well-being, privacy and
dignity of persons accessing abortion clinic.

Necessity

The Victorian and Tasmanian Acts are necessary in that limiting requirements – e.g.
that the protests involve intimidation or harassment – would not provide reasonably
practicable alternatives.

Balancing

The Victorian and Tasmanian Acts are adequate in their balance in that the limited
interference with the implied freedom is not manifestly disproportionate to the
statutory objectives.

What distinguishes Brown and Clubb/Preston?

 In Clubb/Preston, there was greater clarity about the areas within which the protests
were prohibited i.e. 150m.
 In Brown, on-site protests were the primary means of publicising environmental issues
(Clubb v Edwards; Preston v Avery (2019) 267 CLR 171 at [81] per Kiefel CJ, Bell and
Keane JJ).
 In Brown, on-site protests did not involve an attack upon the privacy and dignity of other
people (Clubb v Edwards; Preston v Avery (2019) 267 CLR 171 at [82] per Kiefel CJ,
Bell and Keane JJ).
SECTION 92
s 92 On the imposition of uniform duties of customs, trade, commerce and intercourse
among the States, whether by means of internal carriage or ocean navigation, shall be
absolutely free.

SKELETON

Has two limbs:


1. Freedom of interstate trade and commerce
2. Freedom of interstate intercourse

Freedom of Interstate Trade and commerce


Does the law impose a discriminatory burden on interstate trade of a protectionist sense?
Discrimination can advance from a law on its face or in effect - Cole
- On its face - Cole
- In effect: Market advantage – Castlemaine, Bath, Cole

Is the burden proportionate so as to achieve a non-protectionist legitimate objective?

There are diverging approaches of the court in considering this issue: reasonable necessity
and structured proportionality.

Structured proportionality
A law will not infringe s 92 if it is ‘appropriate and adapted’ to achieving a legitimate state
objective - Castlemaine

Suitability: Does the law have a rational connection with a non-protectionist


legitimate objective
Necessity: compelling alternative means of achieving the same purpose
Balancing: A value judgment balancing the importance of the purpose of the burden
and the extent it restricts free trade.

Reasonable necessity

The majority in Betfair and Gageler J and Gordon J in Palmer favour a reasonable
necessity test. A similar conclusion would be reached in this approach.

Conclude

Freedom of Interstate Intercourse?


Freedom of interstate trade and commerce

Cole v Whitfield (1988) 165 CLR 360 (W [8.40]

Facts
• Tasmanian regulations prescribed minimum sizes for crayfish
• Whitfield owned a crayfish farm
• Whitfield purchased some live crayfish from a farm in SA all above minimum
size prescribed by SA legislation but below minimum size prescribed by Tas
regulations
• Premises inspected and Whitfield charged with possession of 97 undersized
crayfish.
• Argued that the Tasmanian regulations breached s 92

Held
Unanimous decision that Tasmanian regulations did not breach s92

Reasoning
Drawing on ‘history and context alike’ s92’s effect on interstate trade and commerce was
only to guarantee ‘freedom from discriminatory burden’s in the protectionist sense’ (at 395)
The court found unanimously that:
- the Tasmanian regulations did not breach s92 because they were not discriminatory
in a protectionist sense on its face (applied to all crayfish equally) or in effect (no
market advantage to Tas industry)
- Even if there was an advantage, it was not protectionist when the measure was for a
legitimate State purpose (crayfish conservation)
- The purpose of the section is clear enough: to create a free trade area throughout Cth
and to deny to Cth and States alike a power to prevent or obstruct the free movement
of people, goods, and comm across State boundaries.

S92 after Cole v Whitfield


Ask
- Is interstate trade and commerce subject to a burden?
- Is it a discriminatory burden of a protectionist kind (ie is interstate trade being tested
differently to confer a market advantage on local/interstate trade?)

Bath v Alston Holdings Pty Ltd (1988) 165 CLR 411 (W [8.60])
Difficulty in applying the test due to complexity in the industry
Facts
• Business Franchise (Tobacco) Act 1974 (Vic) regulated the sale of tobacco in Vic
by requiring wholesalers and retailers of tobacco in Vic to acquire a licence.
• Wholesalers and retailers paid a licence fee calculated on the basis of the value of
25% of the amount of products sold during the previous licence period.
• Retailers were not required to include in this licence calculation tobacco originally
purchased from a licensed Vic wholesaler.
• Retailers who bought tobacco from interstate paid more for their licence fee than
Vic retailers who only bought tobacco from Vic wholesalers (because of the way
the licence fee was calculated).
• Proceedings brought vs Alston who had no licence to sell the tobacco in Vic that
he bought from Qld.
• Alston argued Act breached s 92
(basically act allowed you to get a cheaper licence to sell tobacco with a percentage of the
amount of tobacco (bought in Vic) that was sold last licence period deducted)

Held
Majority (Mason CJ, Brennan, Deane and Gaudron JJ) found that the legislation was
discriminatory in protectionist sense in form and substance
Minority (Wilson, Dawson and Toohey JJ)

Reasoning
Majority: legislation was discriminatory in protectionist sense in form and substance.
o At 429: ‘A tax upon retailers in respect of their trading in goods may burden
their trade in interstate goods consistently with the guarantee of s 92 only if it
applies equally to the interstate and local goods which the retailers sell; it
cannot lawfully discriminate between them so as to protect the local
goods’.
o It is the effect of tax on transactions in the market which is material. In this
case, the effect is on the supply of goods to that market.
Minority:
• Looked to practical operation & economic effect of the Act as a whole
• The fee on imported tobacco was merely removing an advantage and was not
discriminatory b/c it brought all to the same position and was competitively
neutral.
GOOD FOR COUNTER ARGUMENT FOR ‘MARKET DISADVANTAGE’

Castlemaine Tooheys Ltd v South Australia (1990) 169 CLR 436 (W [8.80])(7:0)

• A law will not infringe s 92 if it is ‘appropriate and adapted’ to achieving a


legitimate state objective.
• What might be a legitimate state objective?

Facts
Amendments to Beverage Container Act 1975 (SA) which imposed a 15c deposit for non-
refillable bottles but only 4c deposit where beer sold in refillable bottles.
These deposits were refundable
Issue
Did the law discriminate against beer brewed interstate in non-refillables? Did it protect SA
from interstate competition?
SA argued that it was about litter control and energy conservation and therefore not
protectionist.
Held
The law was discriminatory in a protectionist sense in relation to interstate trade because it
was not considered to be appropriate and adapted to litter control and energy conservation.

Reasoning
Mason CJ, Deane, Brennan, Dawson & Toohey JJ:
473: ‘legislative measures which are appropriate and adapted’ to a legitimate object will be
consistent with s92 ‘so long as any burden imposed on interstate trade was incidental and not
disproportionate to their achievement.’…the non-incidental or disproportionate burden may
show the true purpose of the law is to impose the impermissible burden.
Gaudron & McHugh JJ: particularly if there exist alternative means involving no or a lesser
burden on interstate trade.

RAA TEST:
Suitability: Does the law have a rational connection with a non-protectionist legitimate
objective
Necessity:
Balancing: A value judgment balancing the importance of the purpose of the burden and the
extent it restricts free trade.

Betfair Pty Ltd v Western Australia (2008) 234 CLR 418 (W [8.120])

Facts
• Betfair Pty Ltd ran a betting exchange out of Tasmania.
• S 24(1AA) of the Betting Control Act 1954 (WA) made it an offence for a
person to bet through a betting exchange.
• S 27D(1) of the Betting Control Act 1954 (WA) made it an offence for a person
in WA or elsewhere to ‘publis[h] or otherwise mak[e] available a WA race field
in the course of business’ without prior approval.
• Betfair’s application for authorisation pursuant to s 27D(1) was denied.
• Betfair challenged the constitutional validity of ss 24(1AA) and 27D(1) on the
basis of s 92.
Held
provisions invalid by operation of s 92 in imposing a discriminatory burden on interstate
trade of a protectionist kind.
Gleeson CJ, Gummow, Kirby, Hayne, Crennan and Kiefel JJ:
Thought it was protecting betting operators acting within WA

[101]-[102]: Proportionality between discriminatory burden and objective of the law –


use test of ‘reasonable necessity’ to decide if regulation is justified
Introduction of the idea of reasonable necessity for the test of proportionality – not the
RAA test

Betfair Pty Ltd v Western Australia (2008) 234 CLR 418 (W [8.120])
• Betfair Pty Ltd ran a betting exchange out of Tasmania.
• S 24(1AA) of the Betting Control Act 1954 (WA) made it an offence for a
person to bet through a betting exchange.
• S 27D(1) of the Betting Control Act 1954 (WA) made it an offence for a person
in WA or elsewhere to ‘publis[h] or otherwise mak[e] available a WA race field
in the course of business’ without prior approval.
• Betfair’s application for authorisation pursuant to s 27D(1) was denied.
• Betfair challenged the constitutional validity of ss 24(1AA) and 27D(1) on the
basis of s 92.
HELD: provisions invalid by operation of s 92 in imposing a discriminatory burden
on interstate trade of a protectionist kind.
Gleeson CJ, Gummow, Kirby, Hayne, Crennan and Kiefel JJ:
[101]-[102]: Proportionality between discriminatory burden and objective of the law –
use test of ‘reasonable necessity’ to decide if regulation is justified.

Betfair Pty Ltd v Racing NSW (2012) 249 CLR 217; Sportsbet Pty Ltd v New South Wales
(2012) 249 CLR 298 (W [8.140])
In dismissing s 92 challenges French CJ, Gummow, Hayne, Crennan and Bell JJ held at [36]
that:
‘Not every measure which has an adverse effect between competitors will attract the
operation of s 92. The “confined area” in which s 92 operates was emphasised in Cole v
Whitfield. Betfair must establish that the fee conditions imposed upon it…were unauthorised
because their practical effect is to discriminate against interstate trade and thereby protect
intrastate trade of the same kind.’

Not discriminatory in a protectionist sense because the fees applied to all operators

Freedom of interstate intercourse

Palmer v Western Australia [2021] 95 ALJR 229

Facts
- Plaintiffs claimed that Directions made under the Emergency Management Act 2005
(WA) (‘the EM Act’) in response to the COVID-10 pandemic or the EM Act itself
burdened freedom of interstate intercourse by impeding free movement into Western
Australia.
- Alternatively, they claimed that s 92’s freedom of trade and commerce was
contravened by the Directions which imposed a discriminatory burden of a
protectionist kind.

Decision
Kiefel, Keane, Edelman applying a structured proportionality approach (reasonable necessity)
Gordon, Gageler reject structured proportionality but no embrace a reasonable necessity test
Found that the Act did not contravene s92

Reasoning (no unanimity)

Kiefel CJ and Keane:


[40] ‘The guarantee of freedom of interstate intercourse may be taken to refer to both
physical movement and communication across State borders, and to be directed to the
circumstance where borders are used as barriers to freedom of movement between States’.
 Both implied freedom of political communication and s 92 freedom are not absolute.
Application of structured proportionality:
 a law restricting the movement of persons into a State is suitable for the purpose of
preventing persons infected with COVID-19 from bringing the disease into the
community.
 Further, the matters necessary to be considered before such restrictions can be put in
place, including with respect to an emergency declaration and the shortness of the
period of an emergency declaration, suggest that these measures are a considered,
proportionate response to an emergency such as an epidemic.
 It requires that the importance of the public health purpose be measured against the
extent of the restriction on the freedom. It must be accepted that the restrictions are
severe but it cannot be denied that the importance of the protection of health and life
amply justifies the severity of the measures.

Edelman J (helpful, sets out structured proportionality test clearly particularly


difficulty of balancing stage):
[265] Structured proportionality makes explicit and transparent the only three independent
grounds upon which a law might be held invalid as contrary to s 92. First, a law will be
invalid if its very purpose is to undermine the freedom guaranteed by s 92. Secondly, a law
will be invalid if its means of achieving its legitimate purpose are not "reasonably necessary",
in the sense that those means burden the freedom guaranteed by s 92 substantially more than
obvious and compelling alternatives which could achieve the purpose of the law to the same
extent. Thirdly, and in absolutely exceptional cases, a law will be invalid if its legitimate, but
trivial, purpose is inadequate to support the extent of the burden placed upon the high
constitutional purpose of s 92.
 The "structure" in structured proportionality is rigid in its refusal to countenance
fictions or hidden grounds for invalidating legislation. As a matter of logic, each
stage of the enquiry also follows the preceding stage.
 Stage 1: The purpose of the legislative provision, in this sense, is the object, goal, or
aim of the law rather than merely the effect of the law. Of course, since purpose or
intention can be inferred from likely effect. a law will only fail at this stage if one of
its very purposes is to achieve that which was proscribed.
 Stage 2: reasonable necessity: assessed according to the availability and obviousness
of means that could achieve the same legitimate purpose to the same extent but
without burdening, or with a lesser burden on, the freedom guaranteed by s 92.
o without the qualification of "reasonableness", a requirement for necessary
means might be misunderstood as a test of the ingenuity of counsel. In Betfair
Pty Ltd v WA, the joint judgment said that the enquiry should be described as
one of "reasonable necessity"
o reasonableness is not a monolithic standard.
o in the context of s 92 the reasonableness threshold means there will be a
margin of appreciation afforded to Parliament before its legislation will be
found to fall outside the boundaries of choice of the means by which to
implement the legislative purpose.
 Stage 3: A foundational principle of the Constitution is representative democracy, which
generally requires that significant policy decisions be left to the branch of government
best suited to make them: the Parliament. However, the description of the s 92 freedom
as "absolute" supports the possibility of invalidity where Parliament puts a necessary but
extreme burden on the subject matter of s 92 in order to achieve a purpose that is trivial.

Gageler J:
• The guarantee of absolute freedom of interstate intercourse differs from …trade
and commerce only to the extent that absolute freedom of interstate intercourse
extends to freedom from discriminatory burdens of any kind.
• Purpose of s92: (identified in Cole v Whitfield) “to create a free trade area
throughout the Commonwealth and to deny to Commonwealth and States alike a
power to prevent or obstruct the free movement of people, goods and
communications across State boundaries” The constitutional purpose derives
from a structural imperative.
• the two limbs overlap. To the extent they overlap, the guarantee of each must be
observed.
• [139] ‘The standard of reasonable necessity so accepted for the purpose of s
92 as expressing the measure of justification demanded by the trade and
commerce limb for a law imposing a differential burden on interstate trade or
commerce, as I have already noted, equally expresses the measure of
justification demanded by the intercourse limb for a law imposing a differential
burden on interstate intercourse’
• Is against structured proportionality however
• my concern about structured proportionality as a tool of legal analysis is
with its rigidity
• the sequencing and linguistic precision of the standardised three-stage
test tends to obscure the purpose for which the overall inquiry is
undertaken. In consequence, it tends to lessen the sensitivity of the
overall inquiry to the constitutional values which underlie the
constitutional freedom protected by the constitutional guarantee at
stake’.

Gordon J: (overlap with Gageler)


• (necessity) the existence of alternative means cannot be conclusive because the
test for s 92 is one of characterisation. To treat the existence of alternative means
as conclusive that s 92 is infringed, in every case, would be an approach that is
too rigid and prescriptive… In the context of s 92, there can be no “one size fits
all” approach.
• Second, to require the balancing stage of structured proportionality as an
additional step of analysis for s 92 would be to introduce a new element that
would be contrary to the foundations and current operation of s 92. As
explained, the freedom in s 92 is absolute but subject to a reservation…
(compare to Kiefel CJ and Keane) The balancing stage of structured
proportionality is not only concerned with identifying the objects of the
impugned law. Rather, the balancing stage of structured proportionality requires
“a value judgment, consistently with the limits of the judicial function,
describing the balance between the importance of the purpose served by the
restrictive measure and the extent of the restriction it imposes on the freedom”
(McCloy…at 195 [2], 218-219 [83]-[87]). Section 92 neither permits nor
requires this further inquiry…And a secondary test – the balancing stage of
structured proportionality – would introduce a degree of values-based decision
making that s 92 not only can avoid…but must. The Court is often not well-
placed to make such value judgments where the nature of trade, commerce and
intercourse is complex, multi-faceted and evolving (See, eg, Betfair No 1 (2008)
234 CLR 418 at 452-454 [12]-[20]).
• [210] On their proper construction, ss 56 and 67 of the Act, in their application
to an emergency constituted by the occurrence of a hazard in the nature of an
epidemic, comply with the constitutional limitation of s 92 of the Constitution in
each of its limbs. They do not impose an unjustified differential burden on
interstate trade, commerce or intercourse in favour of intrastate trade, commerce
or intercourse. They are not discriminatory.
• Is against rigid structured proportionality

Practice Question

Due to concerns about the dwindling quality of bananas, Australia signs and ratifies the Banana
Quality At All Costs Convention (Convention). The Convention’s object is to ‘prevent the dwindling
quality of the world’s banana plantations through disease’ and provides as follows:

Article 4 - All Signatories acknowledge the need to prevent the spread of banana viruses
internationally and domestically and the importance of investigating measures to prevent the spread
of the Banana Bunchy Top Virus

To implement the Convention into Australian domestic law, the Commonwealth passes the Beautiful
& Bountiful Bananas Act 2005 (Cth).The Act provides as follows:

Section 15: ‘It shall be an offence for a “Non-Infected Australian State” to engage in the trade of
bananas with an “Infected Australian State”.

In Section 1:“Infected Australian State” is defined as a “State in Australia which has been notified by
the Minister as having banana crops infected with the Banana Bunchy Top Virus”.

For the 2022 banana season, Queensland is the only Australian State which has not been notified by
the Minister as being an “Infected Australian State”. Due to cyclone damage in Queensland there is a
shortage of bananas during the peak of the banana season. Due to the Beautiful & Bountiful Bananas
Act 2005 (Cth), Queensland retailers and interstate banana wholesalers are not able to trade in bananas
to try and increase the number of bananas available for purchase by Queensland customers.

Bobby Bandana is a banana agent in New South Wales and is charged by the Commonwealth with
trading in bananas with a Queensland retailer in contravention of section 15 of the Beautiful &
Bountiful Bananas Act 2005 (Cth). Bobby has commissioned a report from the Commonwealth
Scientific and Industrial Research Organisation (CSIRO) which indicates that bananas infected with
the Banana Bunchy Top Virus can be treated with pesticides and are readily identifiable. Advise
Bobby whether there are any constitutional arguments he can raise.

Answe
Valid head of power?
Heads of power however are subject to the constitution

Does the Act contravene s92?(s7,24) would qualify those laws that comes within those HoP

i) Does the Act create a discriminatory burden of a protectionist kind


Not discriminatory in effect – Cole
Proportionate? Perhaps but hard to see a protectionist purpose (emphasise RN stage ad
balancing stage and the CSIRO report)

ii) Does the Act burden intercourse between states

This is a Cth law, hard to contravene s92 in trade and commerce aspect. But not impossible
(discussion in Cole v Whitfield) a protectionist object in Cth law.
- Is it T&C, intercourse, or both?
- Seems more T&C, could also be intercourse—the move of goods/people/electronic
waves
- From Palmer, if it comes within both, the law has to comply with both.
On its face, the Banana Aact is burdening trade btw infectory and non-infectory states.
Is this protecting QLD
What is Cth going to argue in response?-not relevantary discriminatory, devastating disease.
Facts to be used to interigate: (to help Bobby)
necessity stage, other things that could be done.
Indicate what Cth could possibly argue, not appropriate to treat with pesticide, must be
sprayed at a particular stage (Cole v Whitfield)
Likely to see Cth is doing it in a protectionist purpose.
Science, determination, legislation based on what?

In terms of
Palmer, how intercourse is being understood.
Gageler J; is there an unjustified differential burden being imposed, is it relevantly
discriminatory, is there a legitimate purpose being served?

Even if it’s supported by the external affairs HoP, it infringes s92, because it’s not
proportionate to the purpose. (report, necessity, balancing stage) Edleman’s judgment
Evidence (iin chapter 8 winterton)
A discussion about Challenging the legisl at the level of directon/legisl?—the legisl, as
challenging the regulatiionn is more of an administration issue, whether the regulations fall
within the umbrella act (but don’t need to focus on that)
INTERGOVERNMENTAL IMMUNITIES
Question: can a parliament of one jurisdiction create laws which will bind, affect an
executive in another jurisdiction? 
This is a limit to certain laws

STEPS:

1.1 Does the Act either expressly or by necessary implication attempt to bind another
level of government?

1.2 If it is taken to bind another level of government is this attempt constitutionally


valid?/ If yes, is the legislation constitutionally valid? does the Act infringe an implied
limit on intergovernmental immunity? (ie Can the Commonwealth Law bind the State(s)
OR can the State Law bind the Commonwealth?)

CTH BINDING THE STATES

Step 1 Does the Act either expressly or by necessary implication attempt to


bind another level of government?
What’s the intention of the legislation?
There is a presumption that statutes are not intended to bind the Crown/body politic unless
this is clearly rebutted (whether expressly or by necessary implication): - Bradken
Consolidated Ltd v BHP Co Ltd (1979) 145 CLR 107.

Express Intention to bind the Crown

Legislation may provide: “This Act shall bind the Crown”

OR

Legislation may provide: “This Act binds the Crown in right of the Commonwealth, of
each of the States, of the Australian Capital Territory and of the Northern Territory…” 

Implied Intention to Bind the Crown (if the Act is silent)

Traditionally, the presumption that had been difficult to rebut when the intention was not
clear on its face. However, where the legislative purpose of the Act would be substantially
frustrated, the presumption may be more easily rebutted – joint judgment in Bropho,
Jacobsen
- Look at the purpose of the legislation and the effect of excluding government from its
sphere of operation. If this demolishes the effectiveness of the act, then the legislation
can be read as having an implied intention to bind the crown – Bropho, Jacobsen
Jacobosen
CTH issuing warrants – hard to enforce the law without entering state departments
Therefore by implication could bind the states
Step 2 Is the legislation constitutionally valid or does the Act infringe an
implied limit on intergovernmental immunity?
*Amalgamated Society of Engineers v Adelaide Steamship Co (1920) 28 CLR 129 (The
Engineers’ case) (W 970)
HELD: State government instrumentalities can be bound by Commonwealth legislation and
the same principle would apply to State legislation subject to constitutional limits (rejection
of doctrine of immunity of instrumentalities).

Cth Binding States

Post Engineers, Cth legislation can bind or affect state and their instrumentalities.

However, the law cannot infringe the Melbourne Corporation principle as refashioned and
collapsed in Austin and Clarke which asks whether the Cth law in its substance and operation
in a significant manner curtails or interferes with the capacity of the states to function as
governments.

Where can the Cth not bind the states:

State prerogative powers + Taxation Laws

Structural integrity doctrine--Protection of States and their capacity to function as


governments
- Melbourne Corporation

Cases on this exception:

Melbourne Corporation v Commonwealth (1947) 74 CLR 31 (W 982).


At 66 per Rich J: ‘Such action on the part of the Commonwealth may be invalid in two
classes of case, one, where the Commonwealth singles out the States or agencies to which
they have delegated some of the normal and essential functions of government, and
imposes on them restrictions which prevent them from performing those functions or
impede them in doing so; another, where, although the States or their essential agencies are
not singled out, they are subjected to some provision of general application, which, in its
application to them, would so prevent or impede them.’

Victoria v Commonwealth (1971) 122 CLR 353 (W 980 )


FACTS: Concerned the constitutionality of the Pay-roll Tax Act 1941-1967 (Cth) and Pay-
roll Tax Assessment Act 1941-1969 (Cth) applying to the wages of State public servants.

HELD: Within scope of s 51(ii) and could bind the Crown in right of the State and did
not go so far as to impair the governmental functions of a State.

Gibbs J at 425: ‘Although in some cases it may be possible to show that the nature of
a tax on a particular activity, such as employment of servants, renders the continuance
of that activity practically impossible, it has not been shown that the tax in the present
case prevents the States from employing civil servants or operates as a substantial
impediment to their employment. That tax has been imposed upon and paid by the
States for nearly thirty years, and it has not been shown to have prevented the States
from discharging their functions or to have impeded them in so doing. They may
have less money available for public purposes because they have to pay the tax,
but that could be said of every case in which a tax is imposed on the States, and
in itself it cannot amount to an impediment against State activity sufficient to
invalidate the tax’.
Queensland Electricity  Commission v  Commonwealth(1985) USEFUL
− Commonwealth legislation required the Arbitration Commission to resolve disputes (and
any other dispute that the Queensland government might be involved in) through a fast-
tracked procedure
−Therefore the legislation only targeted Queensland and the way Queensland resolved
disputes
−Held: all judges found it invalid because it was impermissibly discriminating against the
State of Queensland –this fell beyond Commonwealth power and within the first element of
the exception formulated by Mason J
−Mason J: ‘The principle is now well-established and... consists of two elements’
(1) Prohibition against discrimination which involves placing a special burden or disability on
the States
(2) Prohibition against laws of general application which operate to destroy or curtail the
continued existence of the States or their capacity to function 
o Relationships between the two elements: The second element of the prohibition is necessarily
less precise than the first; it protects the States against laws which, complying with the first
element because they have a general application, may nevertheless produce the effect which it is
the object of the principle to prevent.’

Re Australian Education Union(1995) USEFUL


−Issues: To what extent can Commonwealth awards bind the State public sector? To what
extent can State government employees look to Commonwealth legislation to protect their
entitlements? Is this interfering with the integrity and autonomy of the States by allowed the
Commonwealth to regulate these things?
−Held: federal awards can apply to the State public sector but not in relation to everything –
depends on what particular context you are looking to apply an award to:
Mason CJ, Brennan, Deane, Toohey, Gaudron and McHugh JJ
- Okay for Cth awards to regulate things like minimum wages and working conditions
- Not okay for Cth awards to regulate areas that is critical to that capacity of a State:
the the States have the right to determine redundancies, people it wishes to
employ and terms of this appointment
- Not okay for Cth awards to ‘touch’ higher echelon employees (e.g. laws that are
directed at very senior employees need to be within the control of the States –
ministers, judges, parliamentary officers and top public servants are vital to the State’s
capacity to function as government

Native title Act case (1995)


Mason CJ, Brennan, Deane, Toohey, Gaudron and McHugh JJ (Dawson J dissenting)
o Inconsistency s109
o Upheld the Cth legislation, struck down WA Act
o The Act does not impair what Dawson J described as “the capacity to exercise”
constitutional functions though it may affect the ease with which those functions are
exercised’
Per curiam. 
The requirement that the State pay compensation if it exercises a power of compulsory
acquisition imposed a burden on the exercise of an essential State power, but did so as an
incident of the protection of native title. Section 51(xxvi) is not impliedly limited to preclude
the Commonwealth from protecting the holders of native title in that way.

Austin v Commonwealth (State Supreme Court Judges Superannuation case) Cth law
being challenged
(Cth superannuation legislation treated judges and other state employees as separate from
high earners of the general public - taxed the state directly in the situation rather than the
super companies –
HELD: invalid - undermined the states interest to provide renomination in what it regards
as appropriate to a high-level official
DECISION:
5:1 on law, 1 limb vs 2 limbs (McHuge J)
4:2 on outcome, Cth law invalid vs valid

Gleeson CJ at [24]: In a separate judgment for the majority,

‘The adverse financial impact on the States of the pay-roll tax, or the fringe benefits tax,
both of which were held valid, far exceeded the financial consequences of the laws held
invalid in Melbourne Corporation or Queensland Electricity Commission. It was the
disabling effect on State authority that was the essence of the invalidity in those cases. It
is the impairment of constitutional status, and interference with capacity to function as a
government, rather than the imposition of a financial burden, that is at the heart of the
matter, although there may be cases where the imposition of a financial burden has a
broader significance.’

Gaudron, Gummow and Hayne JJ at [124]:


rejected the separate discrimination limb found in the Melbourne Corporation principle, conflating
the principles into a single principle of structural integrity
‘There is, in our view, but one limitation, though the apparent expression of it varies with
the form of the legislation under consideration. The question presented by the doctrine in
any given case requires assessment of the impact of particular laws by such criteria as
“special burden” and “curtailment” of “capacity” of the States “to function as
governments”. These criteria are to be applied by consideration not only of the form but
also ‘the substance and actual operation’ of the federal law’.
 the majority stressed the importance of judicial remuneration in attracting suitable judges and
securing their independence. They found this tax effectively forced the states to adjust their
remuneration in order to safeguard judicial standards i.e. increase judges pay. For this reason,
they found the law impaired the states' ability to exercise their essential functions.

McHugh J at [224]:
the only judge to endorse the two-limbed Melbourne Corporation principle. On the application of
the first limb (discrimination), he found the Commonwealth law in singling out (and thus
discriminating against) state judges placed a burden upon the states and was thus invalid.
‘Perhaps nothing of substance turns on the difference between holding that there are two
rules and holding that there is one limitation that must be applied by reference to "such
criteria as 'special burden' and 'curtailment' of 'capacity' of the States 'to function as
governments’”. If there is a difference in content or application, it may lead to unforeseen
problems in an area that is vague and difficult to apply. If there are no differences, no
advantage is to be gained by jettisoning the formulation of Mason J in Queensland
Electricity Commission’.
Kirby J at [281] (dissenting as to the result):
agreed with the majority's assessment that the Melbourne Corporation principle is
actually reducible to a one-limbed test.
He found the burden onn states of having judges with higher tax liabilities was not heavy
enough to impair state functioning. The effect was marginal at best.
‘In so far as a difference has emerged in this case between the joint reasons and the
reasons of McHugh J, I agree with the former that the two aspects of the implied
limitation upon federal legislative power, noted in past decisions, are essentially
manifestations of the one constitutional implication. Both are referable to the underlying
conception concerning the nature of the Australian federation. I share the view that each
identified defect is to be determined by reference to the effect of the impugned legislation
on the continuing existence of the States, and whether there is an impermissible degree of
impairment of the State's constitutional functions. The presence of discrimination against
a State may be an indication of an attempted impairment of its functions as
the Constitution envisaged them. But any discrimination against States must be measured
against that underlying criterion’.

*Clarke v Commissioner of Taxation (2009) 240 CLR 272 (W 1006)


French CJ at [34]:
Justice French provided useful criteria to determine if the law significantly affects the
capacity of the states: ‘In my opinion, the application of the implied limitation requires a
multifactorial assessment. Factors relevant to its application include:
1. Whether the law in question singles out one or more of the States and imposes a
special burden or disability on them which is not imposed on persons generally.
2. Whether the operation of a law of general application imposes a particular burden
or disability on the States.
3. The effect of the law upon the capacity of the States to exercise their
constitutional powers.
4. The effect of the law upon the exercise of their functions by the States.
5. The nature of the capacity or functions affected.
6. The subject matter of the law affecting the State or States and in particular the
extent to which the constitutional head of power under which the law is made
authorises its discriminatory application.
Gummow, Heydon, Kiefel and Bell JJ:
o Persons at the higher-levels of gov (i.e members of state legislature) is critical to the state’s
capacity to function.
o Opinions on the limb to be found out (more research required) [65]

Consider the nature of the HoP (French CJ in Clarke) – whether it contemplates impeding
of the states

EG defence power always contemplated states being impeded in times of war therefore
doesn’t make sense to imply this structural limitation

S51(31) (acquisition of property) can contemplate a degree of impairment

S96 – contemplates impeding 2nd Uniform Tax case


STATES BINDING THE CTH

Brief History:

In Uther, the majority upheld the prevailing view since Engineers of a reciprocal relationship
between the Cth and the States in which they were equal partners in federation. Dixon J’s
dissenting judgment in Uther however which stated that the states have no title to legislative
power in relation to the rights and duties between the Cth and its people was adopted in
Cigamatic by the majority, overturning the judgment in Uther.
Henderson’s cases redefines the Commonwealth's immunity from State laws, the broad
approach to the immunity is rejected, in which the majority found that State legislation can
apply to the Cth where the legislation does not impair Cth executive capacities but merely
regulates activities undertaken in the exercise of these capacities. The result of Henderson
impacts not only on the extent of States' power to bind the Commonwealth by legislative
action, but also has implications for the States' power to bind the Commonwealth by
executive action.

TOPIC SENTENCE: The current expression of the law relating to whether States may bind
the Cth comes from the majority judgment in Spence at 107-108 which recognised a reverse
of the Melbourne Corporation principle. This principle asks whether the State law in a
significant manner imposes a disability or burden on the exercise of powers and functions of
the Cth which curtail states capacity to function as a government.

Pirrie v McFarlane(1925)
−Member of the Royal Australian Airforce (e.g. Commonwealth employee) was convicted of
driving in Victoria without a license whilst undertaking air force duties
−Issue: whether State legislation can effectively bind the Commonwealth officer?
−Held: yes(3:2), the Commonwealth employee would be bound by the State
legislation unless it chose to shield itself under s109 inconsistency provision (i.e.
Commonwealth legislation prevails over State legislation)
-Rich, Isaacs JJ (dissent): there was no intention that this legislation would bind the
Crown and even if there was such an intention, Commonwealth employee would not be
bound because it was not possible for State legislation to touch such an important
Commonwealth function (eg defence).

Uther’s Case (1947) :


The state legislation in question was a New South Wales Act which provided for a priority
order of debts in the case of the winding up of a company. Under the legislation, claims for
Commonwealth sales tax and payroll tax were in the class of unsecured debts to be paid after
the payment of other specified debts.

−Held:
The majority held, consistently with previous cases, that, subject to inconsistent
Commonwealth legislation, state laws could validly apply to the Commonwealth.
 it was within the constitutional competence of New South Wales to restrict or abolish
the prerogative right of the Commonwealth in this way.
 this is acceptable because the Cth can always pass legislation under s109 and preserve
its priority if it wants to
Dixon J (Dissenting)
Dixon J rejected the prevailing reciprocal understanding of the relationship between the
Commonwealth and the states and instead developed a doctrine that placed the
Commonwealth in a privileged position.
Although that was ‘necessarily a dual system’, in Dixon J's view it was one in which the
Commonwealth had ‘supremacy’ because of the Constitution's affirmative grant of power to
legislate with respect to specific topics and the paramountcy accorded to federal laws by s
109 of the Constitution.
−The priority of Commonwealth debt derives from the prerogative of the Commonwealth
and therefore it is outside the State’s legislative power
−This is different from a situation where the State can affix particular legal consequences to
particular transactions and the Commonwealth chooses to enter into these transaction
regulated by the State legislation
Commonwealth v Cigamatic (1962):
−Similar factual scenario to Uther’s case –NSW legislation sought to change the priority of
payment of debts
−Held: majority follow Dixon CJ in finding (against Uther) that a State law could not
override the Commonwealth’s prerogative right to prior payment.
‘it is not the plan of the Constitution to grant specific powers to the States over defined subjects’
−However there are issues about the scope of this principle:
- Is Dixon J saying that the prerogative or fiscal rights are exceptions and cannot be
touched by State laws?
- Is he going even broader and saying that the commonwealth is immune from State
laws all together?
- Is the relationship between the Commonwealth and its people the deciding factor

Henderson’s Case (1997)


−Commonwealth Defence House Authority (DHA) had the power to lease premises for its
defence personnel
−H, who owned one of these leased properties, asked DHA if they could have the key to the
premises so they could carry out an inspection under the Residential Tenancy Act(NSW)
−DHA refused to give them the key, arguing that they weren’t bound by this NSW legislation
−Held: the NSW legislation applied to the Commonwealth \
 Dawson, Toohey and Gaudron JJ in their joint judgment formulates a distinction
between (a) State laws that intend to impair the Commonwealth capacities (the rights,
powers, privileges and immunities of the Crown; like prerogatives or fiscal rights) and
(b) State laws which seek to regulate activities undertaken in the exercise of the
Commonwealth’s executive capacities 
o The fundamental principle relied on: Mel Corp.S109.
o The legislative power of the States is an undefined residue which, containing
no such authorisation, cannot be construed as extending to the executive
capacities of the Commonwealth…
 Brennan CJ wrote in a separate judgement, similar approach to the joint judgment,
held that State law cannot modify the Commonwealth's executive power, but that the
Commonwealth may become subject to State law if, in the exercise of its executive
power, the Commonwealth chooses to engage in an activity which is governed by
State law of general application
 McHugh and Gummow JJ rejected the distinction between the capacities of the
Crown and the exercise of those capacities. Their Honours instead took a broad view
of the scope of the Commonwealth's immunity from State laws.
o McHugh J based view on Dixon J in Cigamatic,
 Kirby J: nnaarrowest view of the immunity
o His Honour considered that the Commonwealth's immunity from State laws is
not significantly greater than the States' immunity from Commonwealth laws.
- Ratio: The majority found therefore that as long as the State law doesn’t impair
the executive capacities of the Cth, a general state law can regulate activities that
the Cth chooses to enter
Kirby J (dissent)
−His honour views intergovernmental immunities as less relevant than s109 in this
circumstance
−He believes the distinction formulated by the majority is uncertain, unstable and unhelpful
−He proposes that there needs to be a reversal (doctrine of mutuality) of the Melbourne
Corporation principle applying in this State context –i.e. need to ask whether the State
legislation steps on the ability of the Commonwealth to function as a Commonwealth
government
- Hendersons: State laws affecting the Commonwealth executive
- Melbourne Corporation: State laws affecting the Commonwealth body politic (e.g.
state judges)–therefore Kirby J’s approach allows you to talk about the
Commonwealth’s functioning more broadly

Spence v Queensland (2019) 268 CLR 355

FACTS – Electoral Act 1992 (Qld) and Local Government Electoral Act 2011 (Qld) amended
to stop property developers from making gifts to political parties.
-Commonwealth Parliament introduced s 302CA of Commonwealth Electoral Act 1918 (Cth)
as a response which permitted gifts to parties including if gift might be used towards costs of
voting at federal elections.
HELD:
 s302CA(Cth leg) was found by the majority to be invalid
 Qld Acts were valid and were not outside State legislative power.
 Acceptance of reverse operation of Melbourne Corp principle by majority of
HC

DECISION:
Kiefel CJ, Bell, Gageler and Keane JJ at [107]-[108]:
No return to extreme and discredited notions of inter-governmental immunity is therefore
involved in recognising that the Commonwealth and the States reciprocally have the benefit
of the structural implication recognised in the Melbourne Corporation Case. The reciprocal
application of that structural implication has been, and should remain, constitutional doctrine.
Whilst it is one thing to acknowledge the reciprocal operation of the doctrine of inter-
governmental immunities expounded in the Melbourne Corporation Case as a limitation on
the permissible exercise of State legislative power as well as a limitation on the permissible
exercise of Commonwealth legislative power, it is quite another thing to find that limitation to
have been transgressed. As explained in Fortescue Metals Group Ltd v The Commonwealth,
insofar as it imposes a limitation on the exercise of Commonwealth legislative power,
application of the doctrine “requires consideration of whether impugned legislation is directed
at States, imposing some special disability or burden on the exercise of powers and fulfilment
of functions of the States which curtails their capacity to function as governments”.
Reciprocally, insofar as it imposes a limitation on the exercise of State legislative power,
application of the doctrine requires consideration of whether an impugned State law is
directed at the Commonwealth so as to impose some special disability or burden on the
exercise of powers and fulfilment of functions of the Commonwealth which curtails the
capacity of the Commonwealth to function as a government.

Edelman J at [310]:
‘The assumption upon which the Melbourne Corporation principle is based supports an
implication that constrains any State or Commonwealth power to make a law that is
destructive of the continued co-existence and independence of the Commonwealth and States
as separate governments’. 

at [313]-[316]:
the law's "very object is to restrict, burden or control an activity of the States". However, such
an object is a relevant but not determinative criterion. Distinguish between a law that affects the
capacity of a State to exercise constitutional functions and a law that affects the "ease with which
those functions are exercised". Or, in a formulation which might amount to the same thing, it was said
by three Justices in Austin v The Commonwealth that the curtailment or weakening must be
"significant"

All of these formulations can be reduced to a consideration of the magnitude of the burden
upon the other polity’s capacity to function as a government. That assessment will be one of
evaluation and degree. The magnitude of a burden has dimensions of both breadth and depth.
A burden will be more deeply felt the more that it is targeted at the other polity and the more
essential the governmental function that it curtails is to that other polity…

A burden will be wider the more that it curtails the operation of the governmental functions of
the other polity. The curtailing of the governmental functions of the other polity might
arise merely by curtailing the range of legislative policy choices of the other polity. Even
more significantly, it might curtail the operation of key governmental institutions of the
other polity.

If the magnitude of the burden on the other polity’s capacity to function as a government is
relevant then it should also be relevant to consider the significance of the law for the capacity
of the polity enacting it to govern… (also should consider the impact of the legislation on the
State’s ability to govern)

PRACTICE QUESTION 

As part of the Commonwealth Government’s review of history education in Australia, the


Government has become concerned that parliamentarians may not have a sufficient knowledge of
Australian civics and history. The Commonwealth passed the Parliamentary History Knowledge and
Civics Act 2022 (Cth) which provides as follows:
Section 3: ‘All parliamentarians who fail three times the Australian history knowledge and civics
confirmation test, as set out in the Appendix to this Act, shall be suspended from Parliament for a
period of 3 months’

Section 15: ‘This Act shall bind the States, the Territories and the Commonwealth’

In response, the Western Australian Parliament enacts the Western Australian Knowledge Act 2022
(WA). This Act sets up a rival WA knowledge test which requires a parliamentarian representing
Western Australians as a member of the House of Representatives or the Senate to pass an online
Western Australian knowledge test. Those who fail after two attempts are to be ‘named and shamed’
on the cover of the West Australian newspaper and forced to do a Western Australian knowledge
course at TAFE before re-sitting the test.

The Prime Minister, Scott Morrison seeks your advice as Commonwealth Solicitor-General in
relation to the likely constitutional validity of both enactments.

Apply the Mel corp principle in both ways.


The advice following is for the Prime Minister who presumably would like the Cth enactment
to be valid, and the WA enactment not to be valid.

The Cth Act

The validity of the Cth Act. What we’ve told…whether there’s a HoP to support this. (guess
its Nationhood)

Whether there’s aany limits onn the Cth law.

The intention of the law: intending to apply (s15) to state parliaments. (Don’t rush to Mel)

Then get to the question is the intention valid…(Mel)

The head of power is presumed to be the nationhood power and the act will be valid under
this power unless a limitation applies.

A limit of intergovernmental immunity may apply if (1) the Act intends to bind another level
of government (2) in a manner that is constitutionally invalid.

Cth intention to bind the States


There is a presumption that statutes are not intended to bind the body politics unless this is
clearly rebutted- Bradken. As the Cth Act makes the express statement of intention that the
States will be bound, this presumption is rebutted provided no express or implied limits exist.

Limits: intergovernmental immunities

Post Engineers, Cth legislation can bind state government instrumentalities. However, the
law cannot infringe the Melbourne Corporation principle as refashioned and collapsed in
Austin and Clarke which limits Cth laws from curtailing or interfering in a significant manner
with the capacity of the states to function as governments.

It might be argued that the law doesn’t curtail the function of government because the law
imposes a burden equally amongst the States and the Cth – Clarke and per Re A.E Union the
Cth can regulate some aspects of the public sector and that temporary suspension may be one
such aspect.

On balance however, by suspending parliamentarians for 3 months the Act has a reasonable
probability of significantly interfering in State Parliament processes and therefore their
capacity to exercise their constitutional powers.

Therefore, the legislation is limited by intergovernmental immunity and the State will not be
bound by the Cth Law

The State Act

Is it valid? Is the state law for the POGG of WA? Is there a sufficient connection…simply
acknowledge the law is valid.

Naming and shaming-anaalogy to Austin (Judges ), unlikely to runn for parliament?

Parliamentary sitting

From test in Spence?>Is this

Gubernatorial priviledge tax, argument

Cth may argue that this may seem trivial but…

AEU,

Cth can try and say …but they are parliamentarians, WA is interference with consti
status..the significance of it, responsibilities/prerogatives, gov in charge of…

Edleman in Spence, breath and dept, key gov…

How vital is it for federal politicians to know about WA.

French CJ’s multifactorial test (Clarke) . Thinking whether it’s affecting powers or functions
undertaken…

*remind ourselves to think about the nature of the power—if we think it’s nationhood power,
should it really interfere with the power

This advice assumes that the State law is valid.

The current expression of the law relating to whether States may bind the Cth comes from the
majority judgment in Spence at [107-108] which recognised a reverse of the Melbourne
Corporation principle. The principle in Spence asks whether the State law in a significant
manner imposes a disability or burden on the exercise of powers and functions of the Cth
which curtails its capacity to function as a government.
On the facts the Act doesn’t curtail the capacity of the Cth to function as a government
because it doesn’t disrupt any parliamentary processes or the exercise of Cth constitutional
powers - Spence, Henderson.

Therefore the Act is valid and not limited by intergovernmental immunity.

INCONSISTENCY – S109

Why does s109 exist


Section 109 is designed to deal with the fact that there is potential for conflicting laws in the
Federation
•Commonwealth parliament has limited power... but the States have plenary power –lots of
potential for the State laws to conflict with Commonwealth laws

What is a ‘law of the state/ commonwealth’


•Firstly, there needs to be a valid piece of legislation from both jurisdictions
•Look at the Commonwealth legislation –Is it with respect to a head of power?
Characterisation? Do any constitutional limitations apply? –THEN look to s 109
•Look at the State legislation –plenary power to legislate for the POGG (s 2(3) WA) –THEN
look to s 109

However you don’t need “laws” as such:


•Section 109 can apply to: - O’Sullivan
- Commonwealth or State regulations
- Industrial awards – Ex Parte McLean
- Subordinate legislation
- Commonwealth or State acts authorising an award / regulation

•Section 109 cannot apply to:


- Administrative orders, even if this is made pursuant to a statutory authority (e.g. Air
Traffic Control notices)
- Exercises of prerogative powers or common law -there needs to be a

WHAT DOES INCONSISTENCY MEAN

DIRECT INCONSISTECY

(a) Impossible to obey both laws

Rv Licensing Court(1920)
−State legislation authorised the State referendum
−Cth legislation prevented the State referendum or election on the same day as a Cth
election
−Held: State law is invalid under s109

(b) Denial of rights / privileges (inconsistent but it is possible to obey both)


•When one law takes away or impairs a privilege / right that the other law grants
•It may be possible to obey both laws if you decide to waive the privilege / right you are
otherwise entitled to because it is not directly relating to an obligation
- Where there are Commonwealth and State offences with different penalties
- Where one offence is more onerous than the other offence

Clyde Engineering Co v  Cowburn(1926)


−Commonwealth legislation authorised an award set a particular wage for 48 hours of
work
−State legislation set the same wage for 44 hours of work –i.e. same amount for less time
−The State legislation was affecting the rights of the employer under the Commonwealth
award to get an extra 4 hours of work for the same wage –i.e. Commonwealth was losing
out on the 4 extra hours of work
−Held: State law is invalid under s109 
- Knox, Duffy J(Rich J agreeing)-: the two pieces of legislation may be
inconsistent however obedience may still be possible without disobeying the other

INDIRECT INCONSISTENCY – covering the field test

Ex parte McLean (1930) 43 CLR 472 (W327)


• test can apply even if can comply with both laws
•About whether Cth law ‘control[ling] field exhaustively, exclusively and completely’ (at
483) and not just working alongside State law.

FACTS:
Frederick Firth, a shearer, breached his employment contract by shearing sheep so badly they
were injured, failing to notify his employer and leaving without permission.
Prosecuted under s 4 of Masters and Servants Act 1902 (NSW) for breach of contract.
Section 44 Commonwealth Conciliation and Arbitration Act 1904-1928 (Cth) had different
penalties under Federal award which applied here due to contractual breach.
HELD: State law inoperative even though both laws could operate at the same time
DECISION
Dixon J 483:
- If it appeared that the Federal law was intended to be supplementary to or cumulative
upon State law, then no inconsistency would be exhibited in imposing the same duties
or in inflicting different penalties
- The inconsistency does not lie in the mere coexistence of two laws which are
susceptible of simultaneous obedience. It depends upon the intention of the paramount
Legislature to express by its enactment, completely, exhaustively, or exclusively,
what shall be the law governing the particular conduct or matter to which its attention
is directed. When a Federal statute discloses such an intention, it is inconsistent with it
for the law of a State to govern the same conduct or matter'.

The “covering the field test”


a) What field or subject matter does the Commonwealth Law concern?
b) Does the State Law attempt to enter to any extent upon the same field or subject
matter?

Covering the field is covering the “subject, scope, and evident purpose” – Dickson

O’Sullivan v Noarlunga Meat Ltd (1954) 92 CLR 565 (W333) (regulations for meat export
same field)
FACTS: Commerce (Meat Export) Regulations (Cth) enacted pursuant to the Customs
Act 1901 (Cth) regulated processes for export of meat and required an export permit
and approval for premises used for slaughtering meat for export. Similarly, s 52a
Metropolitan and Export Abattoirs Act 1936 (SA) required a ministerial licence for
premises used to slaughter livestock for export and required person to be ‘a fit and
proper person’.
HELD –
 Statutory majority of Dixon CJ, Fullagar & Kitto JJ (later affirmed by
Privy Council) – s 109 applied to render State law inconsistent when
the fields were the same.
 McTiernan, Webb and Taylor JJ dissented in seeing separate fields
being regulated

Jemena Asset Management Pty Ltd v CoINVEST Ltd (2011) 244 CLR 508
(W354)

FACTS: Awards under Workplace Relations Act 1996 (Cth) required relevant
employers to provide long service leave entitlements to employees. The
Construction Industry Long Service Leave Act 1997 (Vic) required employers
to pay to a trustee (CoINVEST Ltd) a long service leave charge for
construction workers to provide for ‘portability’ of entitlements. Appellants
argued State Act was inconsistent and that they did not need to pay relevant
charge or meet Act requirements.
HELD (Unanimously by French CJ, Gummow, Heydon, Crennan, Kiefel and
Bell JJ): no s 109 inconsistency when State Act was complementary to federal
scheme.
 Cth Award not a law of the Cth but ‘has force and effect of such a law
where so provided by the machinery of a Cth statute’ ([11])
 ‘it is not surprising that different tests of inconsistency directed to the
same end are interrelated and in any one case more than one test may
be applied in order to establish inconsistency for the purposes of s
109. All tests of inconsistency which have been applied by this Court
for the purpose of s 109 are tests for discerning whether a "real
conflict” exists between a Commonwealth law and a State law’ ([42]).
 ‘The mischief which the State Act remedies is that workers in
continuous service in the construction industry will be disadvantaged if
they cannot qualify for long service leave, by reason of the itinerant
nature of their employment. That subject matter, of portable long
service leave benefits in the construction industry, is not covered in the
federal instruments’ ([56]).
 Their Honours highlight ‘The importance of clearly identifying the
field said to be covered exhaustively by a law of the Commonwealth
and correctly characterising a law of a State’ ([58]).
 ‘Whilst the federal instruments deal with all the obligations and
entitlements of employers and employees in respect of the grant of, and
payment for, long service leave, arising in the employment relationship
between employers and employees, they do not deal with, or even
mention, portable long service leave benefits, for workers in
continuous service within the construction industry’ ([59]).

c) Does Cth Law intend expressly or impliedly to cover the field completely,
exhaustively or exclusively

By Express Declaration

Express declaration of exclusivity – Wenn

Express declaration of non-exclusivity – Palmsdale (intention not to cover the field – NOTE
DOESN’T AVOID DIRECT INCONSISTENCY)

Wenn v Attorney-General (Vic) (1948) 77 CLR 84 (and see Dixon J at 120) (W341)

Section 24(2) Re-Establishment and Employment Act 1945 (Cth):


‘The provisions of this Division shall apply to the exclusion of any provisions, providing for preference
in any matter relating to the employment of discharged members of the forces, of any law of a State, or
of any industrial award, order, determination or agreement made or filed under or in pursuance of any
such law, and whether the law, award, order, determination or agreement was enacted, made or filed
before or after the commencement of this section.’
Palmdale-AGCI Ltd v Workers’ Compensation Commission (NSW) (1978) 140 CLR 236, 243 s
99(1) ‘It is the intention of the Parliament that sub-section 21 (2) and (3) shall not, subject to sub-
section (2), apply to the exclusion of a law of a State or Territory.’
Mason J: A Cth statute may provide that it is not intended to make exhaustive or exclusive provision
with respect to the subject with which it deals thereby enabling State laws, not in direct conflict with a
Cth law, to have an operation’.

An express statement that the Cth did not intend to cover the field cannot retrospectively
validate a State law which has previously been invalidated for s109 inconsistency – Metwally
(by 4:3majority)

University of Wollongong v Metwally (1984) 158 CLR 447 (W347)

FACTS:
 Previous decision (Viskauskas v Niland (1983) 153 CLR 280: May 1983) had held that Anti-
Discrimination Act 1977 (NSW) was inconsistent with Racial Discrimination Act 1975 (Cth)
(‘RDA’) when the latter covered the field.
 Cth Parl passed amendment to RDA in 1983 to indicate expressly that it did not, and had never
intended, to cover the field.
s 6A: ‘This Act is not intended, and shall be deemed never to have been intended, to
exclude or limit the operation of a law of a State or Territory that furthers the objects
of the Convention and is capable of operating concurrently with this Act’.
 Metwally case concerned a racial discrimination action brought against the University of
Wollongong in 1981. Could Metwally obtain damages under Anti-Discrimination Act 1977 (NSW)
(in effect, had the Cth amendment revived its operation?). University of Wollongong claimed that
the Cth amendment could not undo the operation of s 109 which had invalidated the NSW Act
previously.

HELD:
MAJORITY: Gibbs CJ, Murphy, Brennan and Deane JJ:
o s6A could apply prospectively (from 19 June 1983) but here trying to give it a retrospective
operation.
o the invalidity of the State law occurred by s 109 of the Constitution and the Commonwealth
Parliament cannot then decide to pass a law to override the operation of the Constitution and
revive the relevant piece of state legislation.
o Metwally could not rely on NSW legislation during the relevant period (1978-81).

MINORITY: Mason, Wilson and Dawson JJ


o Can ‘uncover’ the field by a later law - the Cth Parl can do prospectively what it can do
retrospectively
o did not see s6A as contradicting s109 but rather changing basis for it to operate

By Extent of legislative detail


-the more detailed the Cth law, the more likely it is trying to cover the field see below

O’Sullivan v Noarlunga Meat Ltd (1954) 92 CLR 565


Fullagar J at 591: The Commonwealth regulations ‘constitute an extremely elaborate and
detailed set of requirements…’

Re Residential Tenancies Tribunal of NSW and Henderson; Ex parte Defence Housing


Authority (1997) 190 CLR 410
at 433 (Dawson, Toohey and Gaudron JJ):
‘the Act is neither comprehensive nor exclusive, for in conferring the powers which it does
upon the DHA it assumes an existing legal system within which and by means of which those
powers might be exercised’.
Compared with
Kirby J (dissent) at 498: ‘The fact that federal legislation does not deal elaborately with an
area of law may not evidence an abandonment of that area.. It may simply indicate an
intention to afford the subjects of the federal law an uncontrolled discretion to fix their own
rules, adopting State law as they choose’.

By Nature of the Subject matter

• Is the regulation of that topic only manageable if the Cth law is exclusive?- on

some topics it might be suggested that you can really only have one system of law

operating?

Certain subject matters are more likely to bring about an intention cover the field:
- If the subject matter complies with international obligations( racial discrimination )–
if the enactment operates equally and without discrimination in all states: Viskauskas
- When the topic is only manageable if the Cth law is exclusive – Shipwrecks case
- Areas like bankruptcy or trademarks vs removal of shipwrecks (concurrent authority
is useful and necessary)– Shipwrecks case

A general test? Unresolved issue – might come into play as a more holistic test
When a State law if valid, would alter, impair or detract from the operation of a law of the
Cth Parliament, then to that extent it is invalid – Dixon J in Victoria

IMPACT OF S109 APPLYING

S109 applies so the law becomes inoperative to the extent of the inconsistency and can be
revived once the Cth Act is no longer in operation - Nt Act Case.

You might also like